NEET PG MOCK TEST Merged

You might also like

Download as pdf or txt
Download as pdf or txt
You are on page 1of 103

NEET PG

NEET: JHARKHAND STATE COUNSELLING

MOCK TEST
200 Question with detailed explanation
Anaesthesia
Anatomy
Biochemistry
Community Medicine (PSM)
Dermatology
Forensic Medicine & Toxicology (FMT)
Medicine
Microbiology
Obstetrics and Gynecology (Obgyn)
Ophthalmology
Orthopaedics
Otorhinolaryngology (ENT)
Paediatric
Pathology
Pharmacology
Physiology
Psychiatry
Radiology
2
Surgery
60
NEET PG MOCK TEST

S3
R
EE
R
A
C
Question 1: A worried mother brings her 3-year-old girl who has been complaining about pain
in the joints of her arms and legs for 2 months. On examination, multiple nodules over the bony
prominences suggestive of rheumatoid nodules are noted. The RA factor was found positive in
the investigation, pointing to the diagnosis of RF-positive polyarthritis. Which of the following
complications is not likely to occur in this patient?

Options:
A. Hip joint erosion
B. Atlantoaxial subluxation

60
C. Micrognathia
D. Ankylosis
Correct option: D) Ankylosis

Explanation:

S3
● Ankylosis is not seen in patients with RF-positive Polyarthritis. Polyarthritis is
characterised by inflammation of 25 joints in both the upper and lower extremities. Rheumatoid
factor (RF)-positive polyarthritis resembles the characteristic symmetric presentation of adult
rheumatoid arthritis. Rheumatoid nodules on the extensor surfaces of the elbows, spine, and
over the Achilles tendons are associated with a more severe course and almost exclusively
occur in RF-positive individuals.

1.
R Complications may include
Micrognathia (due to chronic temporomandibular joint disease).
2. Cervical spine involvement, manifesting as decreased neck extension, occurs with a risk
EE
of atlantoaxial subluxation and neurologic sequelae.
3. The hip disease may be subtle, with findings of erosion.

Question 2:
R
A

A child was brought to the paediatrician. On examination, the child's S skull feels soft and
appears as shown below. He is also noted to have delayed dentition and bow legs. What could
be the cause?
C

Options:
A. Scurvy
B. Rickets
C. Osteopetrosis
D. Achondroplasia
Correct option: B) Rickets.
Explanation:
The image shows frontal bossing suggestive of Rickets.
Radiological features are
1. Cupping
2. Splaying
3. Fraying
4. Widening of the provisional zone of calcification (earliest)
5. The white line of calcification (only in healing rickets).

60
6. Biochemical -low Ca -low P4-high PTH high ALP Treatment: Stoss regimen.

Mnemonic "RICKETS"
R-Rachitic rosary
I -Pigeon chest

S3
C-Craniotabes
K-Knock knees
E- End of long bones become wide
T-Teeth-delayed eruption and hypoplasia
S - Skull-Frontal bossing & delayed closure of fontanelles.
R
Question 3: In children, CHF (congestive heart failure) is diagnosed by

Options:
EE
A. Raised JVP
B. Pedal oedema
C. Tender hepatomegaly
D. Basal creps
Correct option: C) Tender hepatomegaly.

Explanation:
R

● Heart failure in infants & children results in some degree of


● hepatomegaly which is usually tender & sometimes splenomegaly.
● Peripheral oedema is age dependent:
A

● In infants, oedema is usually eye & over the flank.


● Older children & adolescents show both periorbital oedema & pedal oedema,
and it occurs late.
C

Question 4: A 8-year-old child presents with a mass in the lumbar region with abdominal pain
with excruciating bone pain. Possible diagnosis is -

Options:
A. Neuroblastoma
B. Wilms Tumour
C. Lymphoma
D. Angiomyolipoma
Correct option: A)Neuroblastoma

Explanation:
This is a case of neuroblastoma that has metastasised
Metastasis is present in 60-70% at the time of diagnosis.
The commonest site of metastasis is the skeletal system, and neuroblastoma
is the most common childhood malignancy that metastasized to bone.

60
Question 5: A child with 11 episodes of diarrhoea in the last 24 hours with sunken dry eyes,
very slow skin pinch, and absent tears came to the Pediatrics OPD. How you will manage the
patient -

Options:

S3
A. ORS solution
B. breastfeeding
C. Start 10% dextrose
D. Start Ringer's lactate
Correct option: D) Start Ringer's lactate.
R
Explanation:
● This is a case of severe dehydration.
● Start IV fluids immediately
EE
● Best IV fluid solution is Ringer lactate
● Normal saline can be used
● Dextrose is not effective
● 100 ml/kg has to be given in the following way as given below-
● AGE <12 months - 30 ml/kg initially for 1 hour, then 70 ml/kg for 5 hours.
● AGE 12 months to 5 yrs- 30ml/kg initially for 30 minutes, then 21/2 hrs.
R

Question 6: 45-day-old infant presents with seizures. Examination reveals he is icteric, having
bulging fontanelles and an opisthotonic posture. Treatment is all except
A

Options:
A. Phototherapy
B. Exchange Transfusion
C. Phenobarbitone
C

D. Chlorpromazine
Correct option: D)Chlorpromazine

Explanation:
Chlorpromazine is not used in hyperbilirubinemia.
Treatment of hyperbilirubinemia includes
1. Pharmacological therapy: Barbiturates (phenobarbitone),
metalloporphyrins (Tin/Sn and Zinc/Zn)
2. Phototherapy
3. Exchange transfusion.

Question 7:

60
A patient presents with recurrent swelling of the face and lips as shown with vomiting and
diarrhoea when emotionally stressed. What is the most likely deficiency?

S3
Options:
A. C1 inhibitor
B. C2
C. C3
D. C1, C2,C4
R
Correct option: A) C1 inhibitors.

Explanation:
EE
● The given clinical scenario is suggestive of hereditary angioneurotic oedema. It is due to
the deficiency of C1 inhibitors. Hereditary angioneurotic oedema is an autosomal dominant
disorder, which is more common than complement deficiency states. Targets of C1 inhibitors are
proteases, specifically C1r and CIs of the complement cascade, factor Xll of the coagulation
pathway, and the kallikrein system.
● A deficiency of C1 inhibitors leads to the unregulated activation of kallikrein. This
R

increases the production of vasoactive peptides such as bradykinin.


● It causes episodes of oedema affecting the skin and mucosal surfaces, such as the
larynx and the gastrointestinal tract. This may result in life-threatening asphyxia, nausea,
vomiting, and diarrhoea after minor trauma or emotional stress.
A

Question 8: The foetal circulation changes to normal circulation at birth with


C

Options:
A. Closure of patent ductus arteriosus
B. Closure of ductus venosus
C. Increased activity of right Ventricle
D. Opening of fossa ovalis
Correct option: A) Closure of patent ductus arteriosus

Explanation:
Increase in Concentration of oxygen in the blood leads to decrease in prostaglandins, causing
the closure of the ductus arteriosus.These closures prevent blood from bypassing pulmonary
circulation, allowing the neonate's blood to become oxygenated in the newly operational lungs.

Question 9: 3 days old newborn with an unknown inborn


error of metabolism, hyperammonemia
in blood.

Options:

60
A. Maple syrup urine disease
B. Urea cycle enzyme deficiency
C. Organic aciduria
D. Phenylketonuria
Correct Option: B) Urea cycle enzyme deficiency.

S3
Explanation:
● Urea cycle enzyme defect
● Catabolism of amino acid leads to free ammonia, which is highly toxic
● Free ammonia is converted into urea by a group of 5 enzymes
● Newborn is usually asymptomatic but later on becomes symptomatic after giving protein.


R Treatment is dietary protein restriction MSUD (maple syrup urine disease)
Defective decarboxylation of branched chain amino acid (leucine, Isoleucine, valine)
● Autosomal recessive
EE
● Smell of maple syrup in urine.
● Phenylketonuria
● Autosomal recessive
● Deficiency of phenylalanine hydroxylase.
● Defect in the conversion of phenylalanine to tyrosine.
● This leads to an increase in the level of phenylalanine.
● This increases phenylalanine converted into phenylpyruvate and phenylacetate.
R

● This phenyl acetate gives a mousy or musty odour in urine/body.


● Sweaty feet odour -Isovaleric academia
● In Alkaptanuria - Urine becomes darkish brown when exposed to air while purplish
A

brown in porphyria.
● Smoky sweet - MSUD
● Mousy or Musty - Phenylketonuria
● Boiled cabbage - Tyrosinemia.
C

Question 10: Child having a long history of haemoglobin


5 gm% next step?

Options:
A. Blood transfusion
B. CBC,reti count with peripheral smear.
C. Start Iron
D. Hb electrophoresis
Correct option: B) CBC,reti count with peripheral smear.

Explanation:
● In the above question, if the child is stable, then no need to give a blood
transfusion. Before starting Iron, we must rule out types of anaemia as Iron is indicated only in
nutritional anaemia.
● Hb electrophoresis is indicated if there is a feature of hemolytic anaemia (thalassemia),

60
so our next step is to complete a hemogram with manual peripheral smear examination.

Question 11: Which one of the following is the most common tumour associated with type I
neurofibromatosis?

S3
Options:
A. Optic nerve glioma
B. Meningioma
C. Acoustic schwannoma
D. Low-grade astrocytoma
Correct option: A) Optic nerve glioma
R
Explanation:
Neurofibromatosis type I (Von-Recklinghewsen disease)
EE
● NF-1 is diagnosed when any two of the following seven signs are present.
1. Six or more cafe-au-lait macules
> 5 mm in prepubertal individuals
> 15 mm in postpubertal individuals
Cafe-au-lait spots are the hallmark of neurofibromatosis and are present in almost 100% of the
patient.
R

2. Axillary or inguinal freckling


3. Two or more Lisch nodules. Lisch nodules are hamartomas located within the iris.
4. Two or more than that, neurofibromas or single plexiform neurofibromas. Typically involves
A

the skin and may be situated along peripheral nerves and blood vessels.
They are small, rubbery lesions with slightly purplish discolouration of the overlying skin.
5. A distinctive osseous lesion.
Sphenoid dysplasia or cortical thinning of long bones.
C

6. Optic glioma
7. A first-degree relative with NF-1

Other findings is
● Pseudoarthrosis of the tibia.
● Scoliosis is the most common orthopaedic problem in NF-1, but it is not specific enough
to be included as a diagnostic criterion.
● Short stature.

Question 12: The following features are true for tetralogy of Fallot, except -

Options:
A. Ventricular septal defect
B. Right ventricular hypertrophy
C. Atrial septal defect
D. Pulmonary stenosis

60
Correct option: C) Atrial septal defect.

Explanation:
Tetralogy of Fallot ( TOF)
● The classical example of cyanotic patients with pulmonic stenosis is tetralogy of Fallot.

S3
● TOF is the commonest congenital heart disease.
● Constituents of TOF
● Ventricular septal defect Overriding or dextroposed aorta.
● Pulmonic stenosis Right ventricular hypertrophy.

Question 13: A 2-year-old boy with normal developmental but delayed speech and difficulty in
R
concentration and communication. He is not making friends. The most probable diagnosis is

Options:
EE
A. Autism
B. ADHD
C. Mental retardation
D. Specific learning disability
Correct option: A) Autism

Explanation:
R

Diagnosis of Autism-
● Delayed speech, difficulty in communication and concentration in a 2-year-old child.
● Autism is characterised by impaired social communication and interaction and repetitive
A

and restricted behaviour.


● These signs all start appearing in children within three years.
● Autism affects information processing in the brain by altering nerve cells and their
synapses' connection and organisation.
C

● It is one of the disorders on the autism spectrum.


● The autism spectrum includes
1. Asperger syndrome.
2. Pervasive developmental Disorder.
3. Delayed speech and difficulty in concentration and communication.

Question 14: Cat bites in child treatment - false is


Options:
A. Cleaning the wound thoroughly
B. Puncture wound most common
C. May require rabies vaccination
D. All of above
Correct option: D) All of above.

Explanation:

60
● A puncture wound is the most common injury from rat and cat bites. Cat bites generally
penetrate deep tissue.
● Infection is the most common complication of any type of bite injury.
● Treatment includes cleansing of the wound, debridement, wound culture, tetanus and
anti-rabies immunisation, and initiation of antibiotics.

S3
● Amoxicillin-clavulanate is an excellent choice for empirical oral therapy for human and
animal bite wounds because of its activity against most strains of bacteria isolated from infected
bite injuries.

Question 15: Cryoprecipitate is useful for managing patients with all the following conditions
except
R
Options:
A. Hemophilia A
EE
B. Von Willebrand disease
C. Hypofibrinogenemia
D. Christmas disease
Correct option: D) Christmas disease

Explanation:
● Cryoprecipitate does not contain factor IX. Hence, it is not useful for managing patients
R

with Christmas disease due to a deficiency of factor IX.


● Components of cryoprecipitate are
1. Factor Vl
A

2. Factor Xl
3. Fibrinogen
4. Fibronectin Von Willebrand factor,
C

Therefore, it can be used in patients with haemophilia A, factor Xlll deficiency,


hypofibrinogenemia, and von Willebrand disease.

Question 16: Evaluation of a patient revealed the presence of a mid-diastolic murmur. JVP
showed prominent a-waves. What is the likely diagnosis? Evaluation of a patient revealed the
presence of a mid-diastolic murmur. JVP showed prominent a-waves. What is the likely
diagnosis?
Options:
A. Mitral stenosis
B. Tricuspid stenosis
C. Mitral regurgitation
D. Tricuspid regurgitation
Correct option: B) Tricuspid stenosis.

Explanation:

60
● The presence of a mid-diastolic murmur and a prominent a-wave in JVP suggest
tricuspid stenosis.
● Mid-diastolic murmurs result from obstruction and/or augmented flow at the mitral or
tricuspid valve level. Thus, both mitral stenosis (MS) and tricuspid stenosis (TS) present with a
mid-diastolic murmur. However, the presence of a prominent a-wave in JVP is pointing more

S3
towards tricuspid stenosis.
● Causes for mid-diastolic murmur:
1. Mitral stenosis
2. Tricuspid stenosis
3. Atrial myxomas
4. Austin flint murmur in severe aortic regurgitation
R
5. Carey-coombs murmur in acute rheumatic fever

● Tricuspid stenosis:
EE
It always occurs in association with MS and is rheumatic in origin.
A. Mitral stenosis
B. Tricuspid stenosis
C. Mitral regurgitation and
D. Tricuspid regurgitation.

● The presence of a mid-diastolic murmur and a prominent a-wave in JVP suggest


R

tricuspid stenosis. Mid-diastolic murmurs result from obstruction and/or augmented flow at the
mitral or tricuspid valve level. Thus, both mitral stenosis (MS) and tricuspid stenosis (TS)
present with a mid-diastolic murmur. However, the presence of a prominent a-wave in JVP is
A

pointing more towards tricuspid stenosis.


● Causes for mid-diastolic murmur:
1. Mitral stenosis
2. Tricuspid stenosis
C

3. Atrial myxomas
4. Austin flint murmur in severe aortic regurgitation
5. Carey-coombs murmur in acute rheumatic fever Tricuspid stenosis: It always occurs in
association with MS and is rheumatic in origin.

Question 17: DNA Methylation is not involved in?


Options:
A. Chromatin remodelling
B. RNA Splicing
C. Gene Silencing
D. Base pair excision
Correct option : D) Base pair excision
Explanation:
● DNA methylation is not involved in the base pair excision repair mechanism.
● DNA methylation is an epigenetic modification used as a gene expression regulation

60
mechanism. It causes gene silencing, plays a role in alternative exon splicing (RNA splicing),
and aids in chromatin remodelling. It, however, Is not involved in the base pair repair
mechanism. Rather it is demethylation which is increased by DNA base pair excision repair.

Question 18: True regarding Mitochondrial DNA is –

S3
Options:
A. One set from each parent
B. It has a higher mutation rate than nuclear DNA
C. It codes for more than 25% of oxidative Phosphorylation proteins
D. It has base pairs
Correct option: B) It has a higher mutation rate than nuclear DNA.


R
Explanation:
Mitochondrial DNA has a 5-10 times higher mutation rate than that of nuclear DNA.
● Option A: Mitochondrial DNA is transmitted by maternal non-mendelian inheritance.
EE
● Option C: It has 37 genes that encode 13 protein subunits of the respiratory chain (a
total of 67, i.e. about 20%).
● Option D: It has 16,569 bp (approximately 1 % of total cellular DNA), while nuclear DNA
has 3.3 billion bp.
● A Mutation in mitochondrial DNA results in MELAS(Mitochondrial Encephalopathy, Lactic
Acidosis, and Stroke), an inherited condition due to NADH-Q oxidoreductase (Complex I) or
cytochrome oxidase (Complex IV) deficiency.
R

Question 19: HMG-CoA in liver mitochondria is inhibited by?


A

Options:
A. Insulin
B. Glucagon
C

C. Glucocorticoid
D. Epinephrine
Correct option: A) Insulin.

Explanation:
● The effect of insulin on ketogenesis is usually associated with the oxidation of excessive
fatty acid (lipolysis), which provides the substrate for ketogenesis. i.e. Acetyl-CoA.
● Factors which inhibit lipolysis →inhibit ketogenesis (and also the production of
HMG-CoA).
● Insulin is an antilipolytic hormone. It inhibits lipolysis and, therefore, ketogenesis. Thus it
decreases the synthesis of HMG-CoA in ketogenesis. So, insulin stimulates as well as inhibits
the production of HMG-CoA.
● Here comes the tricky part of this question. HMG-CoA production in liver mitochondria is
asked in the question
● Ketogenesis occurs in mitochondria.
● Cholesterol synthesis occurs in the smooth Endoplasmic Reticulum and cytosol.

60
● Thus, HMG-CoA production is inhibited by insulin in mitochondria but stimulates it in the
cytosol.

Question 20: Increased uric acid levels are seen in which glycogen storage disease?

S3
Options:
A. Type I
B. Type II
C. Type III
D. Type IV
Correct option: A) Type I


R
Explanation:
Purine overproduction and hyperuricemia in von Gierke disease (glucose-6-phosphatase
deficiency) occur secondary to enhanced generation of PRPP precursor, i.e. ribose-5-phosphate
EE
(a pentose).
● In glucose-6-phosphatase deficiency, glucose-6-phosphate cannot be converted to
glucose.
● Accumulated glucose-6-phosphate is then metabolised via an HMP shunt, generating
large amounts of the precursor of PRPP, i.e. ribose-5-phosphate. The increased synthesis of
PRPP increases the de novo synthesis of purine nucleotides.
R

Question 21: Most important factor which causes lactic acidosis in alcoholics -

Options:
A

A. Production of NADH
B. Formation of acetaldehyde
C. Production of acetate
D. None of the above
C

Correct option: A) Production of NADH.

Explanation:
● Excess intake of alcohol → Excess production of NADH with
a concomitant ↓↓ NAD.
↑↑NADH availability favours the reduction of pyruvate to lactate and oxaloacetate to malate,
decreases its availability for gluconeogenesis, and decreases the synthesis of glucose. This
results in Hypoglycemia.
● Excess of lactate production leads to lactic acidosis.

Question 22: Which of the following elements is known to influence the body's ability to handle
oxidative stress?

Options:

60
A. Fluoride
B. Iron
C. Copper
D. Selenium
Correct option: D) Selenium.

S3
Explanation:
1. The activity of antioxidant enzymes depends on the supply of minerals:
● Manganese
● Zinc
● Copper
● Selenium
2.
R Copper (Cu), zinc (Zn) and manganese(Mn) are required for the activity of superoxide
dismutase.
3. Selenium is required for glutathione peroxidase activity.
EE
Question 23: is HHH syndrome due to a defect in?

Options:
A. Tryptophan metabolism
B. Histidine transporter
C. Branched-chain AA metabolism
R

D. Ornithine transporter
Correct option: D) Ornithine transporter.
A

Explanation:
● Hyperornithinemia, hyperammonemia, homocitrullinuria (HHH) syndrome is an
autosomal recessive disorder of ornithine transport caused by mutations in gene SLC25A15
encoding the ornithine transporter protein (ORNT1).
C

● The defective activity of the ornithine transporter across the mitochondrial membrane
causes a functional deficiency of two mitochondrial enzymes: Ornithine transcarbamylase,
which catalyses the condensation of ornithine and carbamoyl phosphate to citrulline.
● Ornithine-8-aminotransferase (OAT) metabolises the ornithine to pyrroline-5-carboxylate
and, ultimately, glutamate and proline.
● Ornithine accumulates in the cytoplasm, and its deficiency in mitochondria causes a
secondary urea cycle disorder and hyperammonemia.
● Carbamoylphosphate accumulates and undergoes alternative metabolism to form:
1. Homocitrulline - Excreted in urine
2. Orotic acid
3. Plasma.

Question 24: Ninhydrin test is used for?

Options:
A. Bile salts

60
B. Amino acids
C. Nucleic acid
D. Lipids
Correct option: B) Amino acids.

S3
Explanation:
Following are some important chemical reactions.

A. For determining the amino acid sequence in the polypeptide chain, the type of reaction used:
-
● Amino-terminal (N-terminal) of amino acid is tagged with some reagent. splitting it off by
R
hydrolysis and identification of tagged amino acids. Repetition of the same reaction with new N-
terminal of subsequent amino acids and so on. Identification of amino acid sequence by two
reactions:
EE
● Sanger's reaction: - Sanger's reagent ( 1 -fluoro-2,4- dinitrobenzene) is used to tag the
amino terminal of amino acid. Edman's reaction: Uses Edman's reagent (phenylisothiocyanate)
to tag amino-terminal.

B. The type of reaction used for identifying an individual or group of amino acids is frequently
used for qualitative detection and quantitative measurement of various amino acids.
R

● Ninhydrin test: All a-amino acids.


● Xanthoproteic reaction: Aromatic amino acids (Tyrosine, tryptophan, phenylalanine).
A

● Millan's test (Millon-Nasse reaction): Tyrosine(phenol group of tyrosine). Therefore in


tyrosinase -t is positive.
● Aldehyde test: Tryptophan (indole ring)
● Hopkins-cole reaction: Tryptophan (indole ring)
C

● Sakaguch's reaction: Arginine (guanidinium group of arginine).


● Sulphur test: Cysteine (sulfhydryl group)
● Nitroprusside test: cysteine (sulfhydryl group)
● Pauly's test: Histidine (imidazole group)
● Biuret reaction: Peptide bond
● Diazo reaction Q (Pauli's): For detection of Histidine or tyrosine.
Question 25: According to NCEP-ATP III, which among
the following have not been included in metabolic syndrome?

Options:
A. High LDL
B. Hypertriglyceridemia
C. Central Obesity
D. Hypertension
Correct option: A) High LDL.

60
Explanation:
● Metabolic syndrome refers to the co-occurrence of several known cardiovascular risk
factors, including insulin resistance, obesity, atherogenic dyslipidemia, and hypertension. These
conditions are interrelated and share underlying mediators, mechanisms, and pathways.

S3
● Criteria for the Diagnosis of Metabolic Syndrome Elevated waist circumference: (For
men >90 cm and for women, >80 cm).
● Elevated triglycerides: >150 mg/dL
● Reduced HDL (“good”) cholesterol: For men, <40 mg/dL; for women, < 50 mg/dL
● Elevated blood pressure: >130/85 mm Hg
● Elevated fasting glucose: >100 mg/dL


R Insulin resistance (hyperinsulinemia)
Additional parameters include: coagulation abnormalities, hyperuricemia,
microalbuminuria, non-alcoholic steatohepatitis (NASH) and increased CRP.
EE
● Diagnosis is made if any 3 out of the 5 criteria given above.

Question 26: CO acts by inhibiting which component of the respiratory chain?

Options:
A. Cytochrome b
B. Cytochrome C oxidase
R

C. NADH CoQ reductase


D. Oxidative phosphorylation
Correct option: B)Cytochrome C oxidase.
A

Explanation:
● Inhibitors of Electron transport chain (Respiratory chain)
● Complex I:- Barbiturates (amobarbital), piericidin A, rotenone, chlorpromazine,
C

guanethidine.
● Complex II: Carboxin, TTFA, malonate.
● Complex III: Dimercaprol, BAL, actinomycin A, Naphthyloquinone.
● Complex IV (cytochrome c oxidase): Carbon monoxide (CO), cyanide (CN), H2S, azide
(N3-).

Question 27: Refsum's disease is due to a deficiency of which of the following enzyme?
Options:
A. Malonate dehydrogenase
B. Thiophorase
C. Succinate thiokinase
D. Phytanic alpha oxidase
Correct option: D) Phytanic alpha oxidase.

Explanation:

60
Refsum's disease is a rare autosomal recessive disorder caused by a deficiency of phytanic a
oxides / a hydroxylase /Phytanoyl CoA hydroxylase results in accumulation of phytanic acid
due to its decreased oxidation (i.e. hydroxylation at a carbon by fatty acid a hydroxylase).

Question 28: A 41-year-old female underwent total thyroidectomy, and on the third

S3
postoperative day, the patient complained of numbness in her fingers and perioral area. Which
of the following vessels would have been injured during the surgery?

Options:
A. Anterior branch of superior thyroid artery
B. Posterior branch of superior thyroid artery
R
C. Superior branch of inferior thyroid artery
D. Inferior branch of inferior thyroid artery
Correct option: C) Superior branch of inferior thyroid artery.
EE
Explanation:
The above clinical scenario suggests postoperative hypoparathyroidism due to injury to the
superior branch of the inferior thyroid artery. The parathyroid glands are small yellowish-brown
structures lying between the posterior border of the thyroid gland and its capsule. There are two
superior parathyroid glands and two inferior parathyroid glands. The superior branch of the
inferior thyroid artery supplies these. They secrete parathormone, which helps in the regulation
of serum calcium by the release of calcium from bones.
R

Question 29: Among the following, which is the final structure to drain lymphatics from the
stomach?
A

Options:
A. Coeliac nodes
B. Intestinal lymph trunk
C

C. Para-aortic nodes
D. Lumbar lymph trunk
Correct option: B) Intestinal lymph trunk

Explanation:
Among the options, the intestinal lymph trunk is the last structure to drain lymphatics from the
stomach. The intestinal lymph trunk is a major lymphatic channel in the abdomen. It receives
lymphatics from the pre-aortic group of lymph nodes, such as coeliac, superior, and inferior
mesenteric nodes. The intestinal lymph trunk finally drains into the cisterna chyli.

Question 30: A 44-year-old woman presented with hyperacusis. She recently underwent
reconstructive surgery for a fracture of the mandible at the TMJ. Investigations revealed
paralysis of the stapedius muscle. A branch of which of the following nerves is likely to cause
the given condition?

60
Options:
A. Vll cranial nerve
B. VIll cranial nerve
C. V cranial nerve

S3
D. VI cranial nerve
Correct option: A) VII cranial nerve.

Explanation:
● The given scenario suggests injury to the spinal stapedius, a branch of the Villh cranial
nerve (facial nerve). During surgery involving the parotid and mandibular region, injury to the
R
facial nerve can damage the nerve to the stapedius, hence causing stapedial paralysis, which
leads to hyperacusis.
● The tensor tympani and the stapedius are the middle ear muscles innervated by
EE
branches of V and VII cranial nerves, respectively. Both these muscles contract in response to
loud noises and thereby dampen sound. The tensor tympani increases the tension of the
tympanic membrane and reduces the force of vibration.
● Stapedius pulls the stapes posteriorly and reduces the oscillations
● Paralysis of one, or both muscles cause increased sensitivity to the sounds, a condition
known as hyperacusis.
R

Question 31: A 28-year-old woman in labour requested epidural analgesia. Below which level
would you administer the anaesthesia?
A

Options:
A. Upper border of L1
B. Upper border of L3
C. Lower border of L1
C

D. Lower border of L3
Correct option: C) Lower border of L1.

Explanation:
Epidural anaesthesia is given during labour, and the placement is usually preferred below L1 to
avoid injury to the spinal cord. This is because the adult spinal cord terminates at the lower
border of the L1 vertebra.
In children, it usually terminates at the upper border of the L3 vertebrae.

Question 32: A 22-year-old male was brought to the emergency department after a stab wound
in the anterior chest wall. CT scan revealed arterial extravasation from injury to the internal
thoracic artery. The condition causes a decrease in blood supply through all of the following
blood vessels, except

Options:
A. Superior epigastric

60
B. Musculophrenic
C. Anterior intercostal
D. Posterior intercostal
Correct option: D) Posterior intercostal.

S3
Explanation:
● Posterior intercostal arteries are not branches of the internal thoracic artery hence there
would be no decrease in blood supply through these arteries.
● At the 6th intercostal space level, the terminal branches are
1. Superior epigastric artery
2. Musculophrenic artery Sternal branches to Transversus thoracic muscle
3.
4.
R Periosteum of the posterior sternal surface
Sternal red bone marrow
5. Anterior intercostal arteries for the first 6 intercostal spaces.
EE
● Perforating branches supply the pectoralis major by piercing and then supply to the skin
as direct cutaneous vessels.

Question 33: A patient is given a rapid infusion of insulin. Which of the following electrolyte
abnormalities is expected?
Options:
A. Hyperkalemia
R

B. Hypokalemia
C. Hypernatremia
D. Hyponatremia
A

Correct option: B) Hypokalemia.

Explanation:
Rapid insulin infusion results in hypokalemia. Insulin increases the activity of Na-K ATPase in
C

cell membranes so that more K' is pumped into cells. Hence, serum K+ falls. Insulin-glucose
infusions are used to treat hyperkalemia.

Question 34: A 25-year-old woman is anaemic with a haemoglobin concentration of 6 g/dL.


Which of the following changes has occurred in this case, compared to normal?

Options:
A. ↓Arterial Po2, ↓Mixed venous Po2, 2,3-DPG ↑
B. ↓ Arterial Po2, ↓Mixed venous Po2, Normal 2,3-DPG
C. ↓ Arterial Po2,Normal Mixed venous Po2, 2.3-DPG ↑
D. Normal Arterial Po2,Mixed venous po2 ↓ , 2.3-DPG ↑
Correct Option : D) Normal Arterial Po2. Mixed venous po2 ↓,2.3-DPG ↑

Explanation:
● Normal Arterial Po2, Mixed venous Po2 ↓, ↑2,3-DPG are the changes seen in the above
patient.

60
● The oxygen-carrying capacity of the blood is reduced in an anaemic person, but the
arterial Po2 and oxygen saturation are both normal. The decrease in arterial O2 content is
compensated for by an increase in oxygen extraction from haemoglobin, which reduces the Poo
of venous blood.
● The oxygen unloading at the tissue level is enhanced by an increased level of 23-DPG in

S3
an anaemic patient because 2,3-DPG causes a right shift of the oxygen-haemoglobin
dissociation curve.

Question 35: A mouse is engineered to lack a transcription factor necessary for the normal
development of osteoclasts. Which of the following would be reduced in the knock-out animals
R
Compared to normal littermate mice?

Options:
EE
A. Phosphate deposition in trabecular bone
B. Hydroxyapatite levels in bone
C. Osteoblast proliferation
D. Secretion of acid proteases
Correct Option : D) Secretion of acid proteases

Explanation:
R

● Secretion of acid proteases would be decreased in this case. Osteoclasts produce


proteolytic enzymes and several acids necessary for bone resorption. The enzymes digest or
dissolve the organic matrix of the bone, and the acid causes the dissolution of the bone salts.
A

● Other options:
The acidic pH caused by the osteoclasts.

Question 36: A 6-year-old boy was brought to the OPD with craniotabes, rachitic rosary and
C

bowed legs. Diagnosis of rickets was made. To investigate the cause of rickets and to find the
status of vitamin D in the body, which of the following should be measured?

Options:
A. Cholecalciferol
B. 25 hydroxy vitamin D
C. 1, 25 dihydroxy vitamin D
D. 24, 25 dihydroxy vitamin D
Correct Option: B) 25, hydroxy vitamin D

Explanation:
● Measurement of 25 hydroxy vitamin D (calcifediol) provides more accurate information
about vitamin D levels.
● PTH is increased in vitamin D deficiency. PTH is a major stimulus for the
1a-hydroxylase, so there is increased synthesis of the active hormone, 1,25-dihydroxy vitamin
D. Paradoxically, in severe vitamin D deficiency, the levels of this hormone are often normal.

60
Therefore, measurements of 1,25 dihydroxy vitamin D are not accurate reflections of vitamin D
stores and should not be used to diagnose vitamin D deficiency in patients with normal renal
function.

Question 37: After prolonged injection of GnRH in males, which of the following occurs?

S3
Options:
A. ↑ FSH
B. ↓ LH
C. ↑ spermiogenesis
D. None
R
Correct option: B) ↓LH

Explanation:
EE
● Prolonged injection of GnRH in males decreases LH levels.
● Prolonged administration of GnRH or intermittent administration of high doses of GnRH
analogues suppresses the release of gonadotropin, lowers LH and FSH levels and thereby
reduces testosterone production (i.e. chemical castration).
● Mechanism- Inhibition of the replenishment of GnRH receptors so that insufficient
receptors are available for GnRH functioning.
R

Question 38: A patient with a known brain tumour learns that his pituitary stalk has been
affected. Secretion of which of the following hormones is increased after the sectioning of the
pituitary stalk?
A

Options:
A. Prolactin
B. TSH
C

C. ACTH
D. FSH
Correct option: A) Prolactin.

Explanation:
● The secretion of prolactin is increased after the sectioning of the pituitary stalk. Prolactin
is unique among the pituitary hormones because the predominant central control mechanism is
inhibitory, i.e. dopamine inhibits prolactin secretion. Hence, sectioning of the pituitary stalk (Ex:
In head trauma or compressive skull lesions) prevents this inhibition and leads to increased
secretion of prolactin.
● Note: If the pituitary stalk is cut above the pituitary gland, but the entire hypothalamus is
left intact, the posterior pituitary hormones (ADH & Oxytocin) show a transient decrease
because they are initially synthesised in the supraoptic and paraventricular nuclei of the
hypothalamus and are then transported down to the nerve endings to the posterior pituitary

60
gland, requiring several days to reach the gland.

Question 39: A surgeon is worried about the spread of laryngeal carcinoma to the pre-epiglottic
space. Which of the following structures do not border this Space?

S3
Options:
A. Hyoepiglottic ligament
B. Suprahyoid epiglottis
C. Thyroid lamina
D. Thyrohyoid membrane
Correct option: B)Suprahyoid epiglottis.
R
Explanation:
● Suprahyoid epiglottis does not form a boundary of the pre-epiglottic space. The
EE
infrahyoid epiglottis forms the posteromedial border.
● Two potential spaces within the larynx provide an avenue for the spread of
tumours:Pre-epiglottic space and Paraglottic space.
● The pre-epiglottic space is bounded:
1. Superiorly by the hyoepiglottic ligament, Anteriorly by the thyroid and thyrohyoid
membrane, and Posteromedially by the infrahyoid
2. Epiglottis and quadrangular membrane. The paraglottic space is bounded: Laterally by
R

the thyroid
3. Medially by the ventricle and quadrangular membrane,
4. Inferomedially by conus elasticus Posteriorly by the pyriform sinus.
A

Question 40: A 15-year-old boy was admitted to ER with a reduced level of consciousness after
a day of flu-like symptoms, headache, and vomiting. He appeared somnolent with a GCS of 9.
Physical examination revealed nuchal rigidity with no focal neurological deficits. The blood
C

sample was collected and cultured in a blood culture. What is the blood culture ratio of blood to
the reagent in adults?
Options:
A. 1:5
B. 1:20
C. 1:10
D. 1:100
Correct option: C) 1:10

Explanation:
● The blood culture ratio of blood to the reagent in adults is 1:10.
● For Adults, the blood is inoculated into the bottle containing blood culture medium in
blood to broth ratio of 1:10.
● In the case of children, where the amount of blood drawn is little, a ratio of 1:5 may be
achieved.
● Media used: Bile broth or Glucose broth can be used.

60
● Add SPS [Sodium Polyanethol Sulfonate] -to remove the antibiotic effect.

Question 41:

S3
R
The image shows a culture medium with lactose fermenting and non-lactose fermenting
bacteria. Which of the following is not a component of the same?
EE
Options:
A. Taurocholate
B. Peptone
C. Neutral Red
D. Cysteine
R

Correct option : D) Cysteine

Explanation:
● The image shows MacConkey agar.
A

● Cysteine is not a constituent of MacConkey agar. MacConkey agar is a differential


medium. Lactose-fermenting organisms (Eg: E. coli) produce a pink colour, and
non-lactose-fermenting organisms (Shigella sp., Salmonella sp.) do not change the colour.
C

● Contents of MacConkey agar: (Mnemonic- PLANT)


1. Peptone
2. Lactose Aqar
3. Neutral red
4. Sodium taurocholate.
Question 42: A 30-year-old male patient with a history of febrile neutropenia presented with c /o
continuous fever that was not responding to antibiotics. If a blood culture was done in this
patient, the most likely fungal organism that can be isolated is:

Options:
A. Aspergillus Niger
B. Candida albicans
C. Aspergillus fumigatus
D. Mucormycosis

60
Correct option: B) Candida albicans.

Explanation:
The clinical scenario suggests Candidaemia in a Febrile Neutropenia /Immunocompromised
state patient. Most likely, the infection is caused by Candida.

S3
Question 43: A 28-year-old male presented to the OPD with hyperpigmentation and fever for
which he was treated. A few days later, he again presented with a fever which showed a typical
pattern of double temperature rise in 24 hrs. Which of the following is the most likely clinical
diagnosis?
R
Options:
A. Kala-azar
B. Malaria
EE
C. TB
D. Hodgkin's lymphoma
Correct option: A) Kala-azar.

Explanation:
● A double rise in temperature in 24 hrs is seen in the case of Kala-azar.
● L. Donovan, infection of the classic kala-azar ("black sickness") or dumdum fever type
R

occurs in many parts of Asia, Africa and Southeast.


● Vector: Phlebotomus
● Infective Form - Promastigote Kala-Azar Fever: In the early stages of the disease, there
A

is a mild irregular fever becoming intermittent at a later stage and in 20% of cases, pyrexia
shows a double rise in 24 hrs.
● Bone marrow involvement leads to -Anemia, leucopenia, and thrombocytopenia.
● Hypergammaglobulinemia.
C

● Fever and Hyperpigmentation- Hepatosplenomegaly


● In HIV +ve - Hepatosplenomegaly is absent (atypical presentation)
● Malaria:
The classic symptom of malaria: A cyclical occurrence of sudden coldness followed by shivering
and then fever and sweating.
1. Tertian fever: Occurring every two days in P. Vivax and P. ovale infections
2. Quartan fever: Every three days for P. malariae.
3. P. falciparum infection can cause recurrent fever every 36-48 hours or almost continuous
or less pronounced.
● Hodgkin's lymphoma: Pel-Ebstein fever pattern
● Pel-Ebstein fever pattern: Episodic fever at various times of day and in various intervals,
typically consisting of a three to a ten-day period of fever, followed by a three to ten day period
of apyrexia.

Question 44: A patient presented with loose stools, fever and lower abdominal pain. A stool
examination showed the presence of leukocytes, which raised suspicion of Amoebic dysentery.

60
Which of the following statements best describes intestinal amoebae?

Options:
A. They are usually nonpathogenic
B. Cyst is the invasive form

S3
C. They can cause peritonitis and liver abscess
D. Quadrinucleate cyst is an infective form
Correct option: C)They can cause peritonitis and liver abscesses.

Explanation:
● Intestinal amebae, Entamoeba hartmanni, E. coli, E. polecki, and E. nana are


R
non-pathogenic.
E. histolytica is distinctively characterised by its pathogenic potential for humans,
although infection with this protozoan is commonly asymptomatic (causing "healthy carriers").
EE
● Symptomatic amebiasis and dysentery occur when the trophozoites (invasive) invade
the intestinal wall and produce ulceration and diarrhoea.
● Peritonitis can occur, with the liver the most common site of extraintestinal disease.
● The life cycle of the E. histolytica: QUADRI NUCLEATE CYST [Infective Form] ingested
with contaminated food & water Excystation in small intestine / large intestine TROPHOZOITES
form quadrinucleate cyst under unfavourable conditions, like
1. Active feeding & growing stage
R

2. 8 Metacystic Trophozoites are formed.

Question 45: Question 45: To create anaerobiosis, which organism is used


A

Options:
A. Micrococcus
B. Clostridium
C

C. B. anthracis
D. Corynebacterium
Correct option: A) Micrococcus.

Explanation:
● Use of Obligate aerobic micrococcus -Development of anaerobic condition
(anaerobiosis).
● The micrococcus and the anaerobic organism to be cultivated are inoculated into the
same liquid medium.
● During incubation, the Micrococcus creates conditions favourable for anaerobe growth
by gradually utilising the free oxygen.
● After achieving anaerobiosis, the micrococcus dies due to a lack of oxygen, leaving the
anaerobe in pure culture.

Question 46: All culture media are used for antibiotic susceptibility except -

60
Options:
A. Tetrathionate-F
B. Blood agar
C. Chocolate agar
D. Muller-Hinton agar

S3
Correct option: A) Tetrathionate-F.

Explanation: Most widely practised antimicrobial susceptibility test (AST).

● It is affected by many factors like the type of media used.


● As per guidelines, Mueller-Hinton agar (MHA) is the standard medium.

R Few properties of Mueller-Hinton that make it suitable for antibiotic use:

1. Toxins released from bacteria are known to absorb by starch, so they cannot interfere
EE
with the antibiotics.
2. Second, it is a loose agar. This allows for better diffusion of the antibiotics than most
other plates.

Question 47: Important mechanisms of gene transfer in bacteria are all given in the options
except?
R

Options:
A. Lateral gene transfer
B. Conjugation
A

C. Vertical gene transfer


D. Horizontal gene transfer
Correct option: C) Vertical gene transfer.
C

Explanation:
Gene transfer refers to the process by which genetic material (e.g. DNA)is transferred among
two organisms where the donor sends, and the recipient receives the genetic material.

There are two processes of gene transfer:


1. Horizontal gene transfer (HGT) or lateral gene transfer (LGT)-
Horizontal gene transfer is the process by which genetic material is passed between two
different organisms,i.e. organisms of different species. The recipient is not the offspring of the
donor.

The most important example is gene transfer between the two bacteria.
The processes involved in horizontal gene transfer in bacteria are
1. Transduction
2. Transformation
3. Conjugation

60
2. Vertical gene transfer(VGT) - Vertical gene transfer is the process by which genetic material is
transferred to the organism of the same species, i.e. the donor receives genetic material from its
ancestor. This process is not important in the case of bacteria.

S3
Question 48: Superantigen causes

Options:
A. Polyclonal activation of T-cells
B. Stimulation of B cells
C. Enhancement of phagocytosis
R
D. Activation of complement
Correct option: A) Polyclonal activation of T-cells.
EE
Explanation:
Superantigens can activate up to 20% of the peripheral T-cell pool, whereas conventional
antigens activate < 1 in 10 000.

Question 49: Hemorrhagic fever is caused by

Options:
R

A. West-Nile fever
B. Sandfly fever
C. Ebola virus
A

D. All of the above


Correct option: C) Ebola virus.

Explanation:
C

Ebola virus belongs to hemorrhagic fever (fact).

Question 50: A 27-year-old homosexual black male presented to the clinic with c/o
long-standing tender rash on his right hand. The rash continued to enlarge and became pruritic
even after initial treatment. The doctor sent him for VDRL testing. What type of antigen-antibody
reaction is seen in VDRL?
Options:
A. Slide agglutination
B. Tube agglutination
C. Slide flocculation
D. Tube flocculation
Correct option: C) Slide flocculation

Explanation:
VDRL is a slide flocculation test .

60
Question 51: Secondary hemochromatosis is associated with which of the following:
a. Thalassemia
b. Sideroblastic anaemia

S3
c. Multiple blood transfusions
d. Paroxysmal nocturnal hemoglobinuria

Options:
A.a,b,c are true
B.a,b,d are true
R
C.a,c,d are true
D.Only B, C are true
Correct option: A) a,b,c are true.
EE
Explanation:
Anaemia caused by inadequate hematopoiesis is linked to abnormal increases in iron
absorption from the gut, which can lead to iron excess (secondary hemochromatosis) and harm
to the heart and endocrine organs.
Secondary hemochromatosis is seen in
1. Thalassemia
R

2. Sideroblastic anaemia
3. Multiple blood transfusions.
A

Question 52: A 79 years old male presented with anaemia with splenomegaly. PBS shows
teardrop-shaped cells, and bone marrow examination was normal. The diagnosis is

Options:
C

A. Myelofibrosis
B. Iron deficiency anaemia
C. Folic acid deficiency
D. CML
Correct option: A) Myelofibrosis

Explanation:
Teardrop-shaped cells are seen in myelofibrosis.

Question 53: A 8 Yr old girl child, a known case of Non-Hodgkin's lymphoma on


Cyclophosphamide, now presents with sudden onset fever and petechiae. Bone marrow
examination revealed Hypercellular Lymphoblastic cells. Which of the following Lymphoid
malignancy described here arises from thymic cells?

Options:
A. Hairy cell leukaemia

60
B. Burkitt lymphoma
C. Pre-B cell ALL
D. Pre -Tcell ALL
Correct option: D) Pre- T-cell ALL

S3
Explanation:
● Leukaemia/lymphomas (ALLs) are neoplasms composed of immature B (pre-B) or T
(pre-T) cells, which are referred to as lymphoblasts→present in Leukaemia/ Lymphomas (ALLs)
type of neoplasms.
● About 85% are B-ALLs, which typically manifest as childhood acute “leukaemias.”
● In adolescent males, the T-ALLS present as thymic “lymphomas.”
R
ACUTE LYMPHOBLASTIC LEUKAEMIA:
● Most common type of leukaemia in children.
EE
● Usually presents with sudden onset of Fatigue/fever/petechiae with
Hepatosplenomegaly and /or Lymphadenopathy and Sternal tenderness(+).

Question 54: A patient with Parkinsonism, Mr Rahul, noticed that the therapeutic effect of
levodopa decreased when his physician gave him another drug, but no drug interaction was
seen when he switched to a levodopa-carbidopa combination. What drug is most likely
prescribed by his physician?
R

Options:
A. Metoclopramide
A

B. Vitamin B complex
C. Chlorpromazine
D. Isoniazid
Correct option: B) Vitamin B complex
C

Explanation:
● Pyridoxine is a vitamin B complex.
● The activity of the enzyme dopa decarboxylase is stimulated by the administration of
vitamin B complex, as Pyridoxine is a cofactor for this enzyme.
● Dopa decarboxylase converts levodopa to dopamine. Increased formation of dopamine
in the periphery is undesirable because it cannot cross the blood-brain barrier. Therefore,
stimulation of dopa- decarboxylase → therapeutic effect of l-dopa ↓↓.
● If the enzyme dopa decarboxylase is already inhibited with carbidopa, there will be no
interaction with pyridoxine.

Question 55: A person who came into the emergency with complaints of breathing difficulty for
4 days duration and you admit him seeing that he requires ventilatory support. About 6 days
after ventilation, you notice that the patient I developed new-onset fever spikes and order a

60
CBC, and send the endotracheal aspirate for culture/sensitivity. There is significant I
leukocytosis, and the C/S turns out to be MDR Acinetobacter baumanni. You, being in charge,
would begin treatment with?

Options:

S3
A. Quinupristin and dalfopristin
B. Colistin
C. Lavendamycin
D. Tedizolid
Correct option: B) Colistin
R
Explanation:
● The patient has developed Ventilator-associated Pneumonia (VAP).
● The preferred treatment for this multidrug-resistant organism is Colistin.
EE
● Only 2 forms of polymyxin are in clinical use today: Polymyxin B – Parenteral and
ophthalmic topical.
● Polymyxin E (Colistin):
● Available as a prodrug
● IV /inhaled via nebuliser
● Used as salvage therapy for multidrug-resistant infection
● S/E: Nephrotoxicity Neurotoxicity
R

● Dalfopristin/Quinupristin- Reserved for treatment of severe vancomycin-resistant


Enterococcus faecium (VRE) in the absence of other therapeutic options.
A

Question 56: A 56-year-old patient complained of chest pain radiating to the left shoulder, of a
squeezing type, as described. ECG revealed no specific changes. You suspect it could be an
episode of angina. Which of the following rapidly acting medications can be given in treating this
case through the inhalation route?
C

Options:
A. Amyl nitrite
B. Isosorbide mononitrate
C. Nitroglycerin
D. Propranolol
Correct option: A) Amyl nitrite
Explanation:
1. Amyl nitrite is given by inhalation route and used in cyanide poisoning.
2. Advantages of inhalation route -
● Very rapid absorption.
● Avoids the hepatic first-pass effect.
3. Due to the unpleasant odour and extremely short duration of action, amyl nitrite is now
obsolete for angina.
4. Amyl nitrite is a highly volatile liquid in fragile glass Ampules. Ampules must be crushed

60
with fingers, and the vapours must be inhaled through a protective covering.
5. The antidote for cyanide poisoning is Hydroxocobalamin, a form of vitamin B12, which
also has a very high affinity for cyanide and combines with it to form cyanocobalamin. This
non-toxic compound is excreted in the urine.

S3
Question 57: A 48-year-old female initially presented to the Emergency Department
Complaining of subjective fevers and fatigue for the past 4 days along with other symptoms like
headaches, but not chills, rigour, or chest pain. As the patient had recently travelled from an
endemic region, chances of malaria are high. If this patient develops Complicated falciparum
malaria, which drugs will be used for management?
R
Options:
A. Quinine
B. Artemether
EE
C. Artesunate
D. Hydroxychloroquine
Correct option: C) Artesunate

Explanation:
1. ARTEMISININS:
● Artemisinin and its three major semisynthetic derivatives in clinical use
R

Dihydroartemisinin, Artemether and Artesunate are potent and Fast acting Antimalarials. They
are used for the treatment of Severe P. falciparum malaria and asexual.
● Erythrocytic stages of P.vivax. ARTESUNATE – All patients with Severe Falciparum
A

Malaria should be treated with I/V or l/M Artesunate (at least 24 hours).
2. ARTEMETHER – is used only if Parenteral Artesunate is not available.
3. PROPERTIES:
● Fastest acting Antimalarial drug
C

● Effective against MDR parasites


● Short acting
● C/l in the 1st Trimester.

Question 58: All of the following are features of metoprolol in comparison to propranolol
EXCEPT
Options:
A. It is ineffective in suppressing muscle tremor
B. It is safer in diabetics
C. It is less likely to cause bradycardia
D. Raynaud's disease is less likely to be worsened by it
Correct Option: C) It is less likely to cause bradycardia.

Explanation:
Metoprolol is a cardio-selective Blocker and thus is safer than nonselective blockers

60
(propranolol) in diabetics, asthmatics and peripheral vascular disease patients (e.g. Raynaud's
disease).

Question 59:All are true regarding Isavuconazonium sulphate except:-

S3
Options:
A. Effective against yeasts, moulds, and dimorphic fungi
B. Acts through blocking cytoplasmic proteins
C. Approved for aspergillosis and mucormycosis
D. Devoid of nephrotoxicity
Correct option:
R
Explanation:
● ISAVUCONAZONIUM SULPHATE inhibits L-14 alpha demethylase and inhibits cell wall
EE
formation and does not act through blocking cytoplasmic proteins.
● It is effective against yeasts, moulds and dimorphic fungi.
● It is approved for aspergillosis.

Question 60: A 32-year-old female, Lily, presents to your office with fatigue, muscle weakness
and headache. His blood pressure is 170/120 mm Hg, and her heart rate is 100/min. Laboratory
R

investigation reveals hypokalemia, metabolic alkalosis and ↓↓ plasma renin activity. A mass was
noted on the left suprarenal gland on a CT scan. The patient has prescribed a drug for a few
weeks, and their symptoms subsided. Laboratory values and BP came back to normal values.
A

Which drug is likely to be given to this patient?

Options:
A. Clonidine
C

B.Propranolol
C.Hydrochlorothiazide
D. Spironolactone
Correct option: D)Spironolactone.

Explanation:
● Most likely diagnosis in this patient is aldosterone secreting tumour
(adenoma) leading to primary hyperaldosteronism (Conn's Syndrome).
● Aldosterone excess will cause hypertension, hypokalemia, metabolic alkalosis and
decreased renin.
● Aldosterone antagonists such as eplerenone or spironolactone can be used as medical
therapy for Conn's syndrome.

Question 61: A patient is currently in your rural clinic without much infrastructure for a condition
'x and cannot swallow food/ medications. Considering the condition 'x' may be anything, which
one of the following cannot be used since no intranasal preparation is available?

60
Options:
A. Theophylline
B. Ipratropium bromide
C. Budesonide

S3
D. Terbutaline

Correct option: A) Theophylline.

Explanation:
● The inhalation route can administer beta 2 agonists like salbutamol and terbutaline.


R Inhalational anticholinergic agents - Ipratropium and tiotropium
Inhalational steroids - budesonide, fluticasone, beclomethasone and flunisolide.
● The inhalation route does not use theophylline. It is given by Oral route.
EE
Question 62: Which of the following statements is false regarding phenobarbitone?
a) It acts primarily at the GABA: BZD receptor -chloride ion channel complex
b) It can cross the placenta
c) Contra-indicated in Acute intermittent porphyria
d) It has no significant drug interactions with warfarin.
R

Options:
A . a only
B. a and b only
A

C. a,b and c
D. d only
Correct option: D) d only
C

Explanation:
● Phenobarbitone induces warfarin metabolism.
● Phenobarbitone and primidone act as anticonvulsant drugs due to GABA mimetic as well
as GABA facilitatory properties.
● These drugs increase the duration of the opening of chloride channels. These drugs are
useful in generalised tonic-clonic seizures (GTCS) and partial seizures.
● These drugs are highly sedating, but tolerance develops to this effect. Phenobarbitone is
a powerful enzyme inducer that increases several drugs' metabolism, including warfarin.
● C/I in acute intermittent porphyria.
● Phenobarbitone is the drug of choice for GTCS in infants but can cause hyperkinesia in
older children.

Question 63: Which of the following drugs may enhance the effect of warfarin and increase the
risk of bleeding?

60
Options:
A. Phenobarbitone
B. Ketoconazole
C. Rifampicin
D. Carbamazepine

S3
Correct Option: B) Ketoconazole.

Explanation:
Of the given drugs, ketoconazole is the only microsomal enzyme inhibitor, and it can increase
the effect of warfarin (causing an increased risk of bleeding).
R
Question 64: A 60-year-old male with a previous history of stroke, under the treatment of
Dabigatran and aspirin, was brought to the ED by her daughter as she consumed 15 tablets of
the dabigatran after a fight in the household. You, the doctor-in-charge at the hospital, would
EE
insist on giving which of the following to reverse the effects of the drug?

Options:
A. Rituximab
B. ldarucizumab
C. Trastuzumab
D. Protamine sulphate
R

Correct option: B) Idarucizumab.

Explanation:
A

● Idarucizumab is a humanised monoclonal mouse antibody fragment directed against


dabigatran. The antibody binds dabigatran with an affinity 350-fold higher than dabigatran for
thrombin, and the kidneys clear the essentially irreversible idarucizumab-dabigatran complex.
● If specific reversal agents are unavailable, prothrombin complex concentrate, activated
C

prothrombin complex concentrate, or recombinant factor Vlla have been recommended to


manage patients taking dabigatran, rivaroxaban, apixaban, or edoxaban who present with
life-threatening bleeding, such as intracranial or pericardial bleeding.
● In patients taking dabigatran with serious bleeding in acute renal failure, hemodialysis
can remove dabigatran from the circulation. Dialysis is of no value for removing rivaroxaban,
apixaban or edoxaban because of their higher protein binding.
Question 65: During the internship, you notice a patient is brought to the ED after an overdose
with a drug 'x' and the resident on duty, after collecting history, instructs the nurse to give
N-acetyl-cysteine as mentioned in the chart. NAC is an antidote for?

Options:
A. Heparin
B. Acetaminophen
C. Morphine
D. Benzodiazepine

60
Correct option: B)Acetaminophen.

Explanation:
Paracetamol (Acetaminophen)
● It is metabolised to N-acetyl-p-benzoquinone imine (NAPQI) by microsomal enzymes.

S3
● When a very large dose of paracetamol is taken, glucuronidation capacity is saturated;
more of the minor metabolite is formed, leading to the depletion of glutathione stores.
● This metabolite has a high affinity for sulfhydryl groups and can combine with enzymes
and other bio-molecules resulting in hepatotoxicity characterised by hepatic necrosis.
● The toxicity of acetaminophen can be decreased by sulfhydryl donors like
N-acetylcysteine (antidote of choice).

R Note: Chronic alcoholism induces CYP enzymes, especially CYP2E1, which is involved
in the conversion of acetaminophen to the toxic metabolite NAPQI.
EE
Question 66: Absorption of which of the following drugs increases with fatty meals?

Options:
A. Griseofulvin
B. Amphotericin B
C. Nimesulide
D. Azoles
R

Correct option: A) Griseofulvin

Explanation:
A

Drugs whose absorption increase with fatty meals are


● Griseofulvin
● Statins
● Erlotinib
C

● Lumefantrine
● Albendazole

Question 67: In cases of hyperthyroidism, most of the effects mediated by the increased
hormone levels is due to peripheral conversion of T4 to T3. Which of the following medications
inhibit 5 deiodinase?
Options :
A. Propylthiouracil
B.Methimazole
C. Lugol's iodine
D. Radioactive iodine
Correct option: A)Propylthiouracil.

Explanation:
● For the peripheral conversion of T4 to T3, the enzyme needed is 5'-deiodinase

60
(deiodinase type 1 and 2). It is inhibited by peripheral conversion inhibitors, which are:
1. Propranolol
2. Propylthiouracil
3. Prednisolone
● Secretion inhibitors are

S3
1. NAI
2. Lugol iodine
● NIS Inhibitors are
1. Perchlorate
2. Thiocyanate
● Thyroid peroxidase inhibitors are
1.
2.
R Carbimazole
Propylthiouracil
● Thyroid destroying drugs
EE
1. Iodine 131

Question 68: A 47-year-old female came in with complaints of increased frequency of urination,
especially at night, apart from the sensation of residual urine even after urinating. You order a
few tests, and since they turned out to be normal, you make a diagnosis of benign prostatic
hyperplasia and plan to give medications for the same. When you give the patient the medicine,
you mention that it is a drug that acts on the smooth muscle cells of the prostate, leading to
R

'apoptosis.' Which of the following acts by this mechanism?

Options:
A

A. Doxazosin
B. Terazosin
C. Tamsulosin
D. Both A and B
C

Correct Option: D) Both A and B .

Explanation:
● Induction of apoptosis in prostate smooth muscle cells in BPH is shown by Terazosin
and Doxazosin.
● This apoptosis may lessen the symptoms associated with chronic BPH by limiting cell
proliferation.
● The apoptotic effect of terazosin and doxazosin appears to be related to the quinazoline
moiety rather than 1 receptor antagonism.
● Tamsulosin, a non-quinazoline 1 receptor antagonist, does not produce apoptosis.

Question 69: Xenon, an inert gas, has been considered for use as an anaesthetic agent and
has all the following properties except?
Options:
A. Slow induction and recovery
B. Non-explosive

60
C. Minimal cardiovascular side-effects
D. Low blood solubility
Correct option: A) Slow induction and recovery.

Explanation:

S3
Advantages of Xenon Anesthesia
1. Inert (probably non-toxic to liver and kidney with no metabolism)
2. Minimal effect on CVS function
3. Lowest blood solubility (Lowest blood gas partition coefficient), therefore rapid induction and
recovery.
4. Does not trigger malignant hyperthermia
R
5. Environmental friendly
6. Non-explosive
EE
Disadvantages of Xenon Anesthesia -
1. High cost
2. Low potency (MAC = 70%)
3. No commercially available anaesthesia equipment.

Question 70: Halothane is an inhalational anaesthetic agent that is not flammable. All of the
following are true regarding it, except?
R

Options:
A. Unsuitable for the paediatric population
A

B. Potentiates competitive neuromuscular blockers


C. Hepatitis may occur
D. Contraindicated in patients with cardiac arrhythmia
Correct option: A) Unsuitable for the paediatric population.
C

Explanation:
● It is a good anaesthetic but a very poor analgesic agent.
● On repeated use - hepatitis may occur.
● Dantrolene is used for the treatment of halothane-induced malignant hyperthermia.
● Post-anaesthetic chills and shivering may also occur due to halothane use.
● Pethidine is the treatment of choice for this condition.
● Halothane relaxes the uterus. Due to this property, in the internal version and manual
removal of the placenta, it is the agent of choice. However, due to its uterine relaxing property, it
is contraindicated in labour, because if postpartum haemorrhage results, it will be difficult to
control (contraction of the uterus stops bleeding after labour).
● lt sensitises the heart to the arrhythmogenic action of catecholamines. It is, therefore,
contraindicated in patients with pheochromocytoma.
● It is also a cardiac-depressant drug that causes hypotension, bradycardia and
arrhythmias.
● Due to its bronchodilator action - it is the inhalational agent of choice in bronchial

60
asthma.
● For induction in children -it is an excellent agent.

Question 71: Are Cytokeratin 8 and 18 and ubiquitin present in all except?
Options:

S3
A. Alcoholic hepatitis
B. Non-alcoholic steatohepatitis
C. Primary biliary cirrhosis
D. Acute viral hepatitis
Correct option: D) Acute viral hepatitis
R
Explanation:
● Cytokeratin 8,18 and ubiquitin presence is found in MALLORY DENK BODIES.
● Eosinophilic inclusion bodies are seen in Hepatocyte swelling.
EE
● Tangled skeins of intermediate filaments
● Commonly seen in
1. Alcoholic hepatitis
2. Non-alcoholic steatohepatitis
3. Metabolic disorders like Wilson's disease,
4. Chronic cholestatic disease
5. HCC,
R

6. Primary biliary cirrhosis.

Question 72: A 72-year-old male Rahul with advanced visceral cancer, dies of extensive
A

myocardial infarction. Sterile non-destructive vegetation along the mitral leaflet edges is also
revealed under autopsy. This patient's vegetations pathogenesis is most similar to that of

Options:
C

A. Hypercalcemia of malignancy
B. Distant metastases
C. Trousseau syndrome
D. Raynaud's phenomenon
Correct option: C) Trousseau syndrome .

Explanation:
● The pathogenesis of non-bacterial thrombotic endocarditis (NBTE) often involves
hypercoagulability resulting from the procoagulant effects of the circulating cancer products; the
resulting heart valve vegetation can also be referred to as endocarditis marantic.
● The pathophysiology of NBTE is similar to that of Trousseau's syndrome (migratory
thrombophlebitis), which can also be induced by disseminated cancers such as mucinous
adenocarcinoma of the pancreas and adenocarcinoma of the lungs, possibly related to the
procoagulant effect of circulating mucin.
● Cancer metastases in the heart usually affect the pericardium or myocardium. Valve
metastases are less common and would likely have shown invasive features on histological

60
examination.

Question 73: A 56-year-old female who has not seen a doctor in more than twenty years now
came to OPD with left breast pain. On physical examination, the left breast is markedly
erythematous and swollen. On palpation, the breast is warm to the touch and tender with

S3
significant dimpling of the breast (peau d'orange), and the left nipple is completely retracted.
Most likely diagnosis among the following given options ?

Options:
A. Granulomatous mastitis
B. Micropapillary carcinoma
R
C. Fibrocystic disease of the breast
D. Inflammatory carcinoma
Correct option: D) Inflammatory carcinoma.
EE
Explanation:
● The presentation is that of inflammatory carcinoma of the breast.
● It has an extremely poor prognosis.
● The name "inflammatory" is a misnomer, as typically, no inflammation is present.
● The underlying carcinoma typically does not form a discrete palpable mass and is
usually diffusely infiltrative.
R

● In this type, cancer cells have invaded the skin and suspensory ligaments of the breast
→causing dimpling and distortion of the normal breast architecture.
A

Question 74: A 8-year-old girl child developed a 10 mm area of induration on the left forearm
72 hours after intradermal injection of 0.1 ml of purified protein derivative (PPD). Most likely, the
X-ray of this patient shows?
C

Options:
A. Marked hilar adenopathy
B. Upper lobe calcifications
C. No abnormal findings
D. Reticulo-nodular densities
Correct option: C) No abnormal finding.
Explanation:
● Most Mycobacterium tuberculosis infections are asymptomatic and subclinical infections.
● In most healthy individuals, primary tuberculosis is asymptomatic, although it may cause
fever and pleural effusion.
● The only evidence of infection, if any, remains → a tiny, fibrocalcific pulmonary nodule at
the site of the infection.
● In such lesions, viable organisms may remain dormant for decades.
● If immune defences are lowered, the infection may be reactivated, producing
communicable and potentially life-threatening diseases.

60
● Test for latent tuberculosis - Detection of T-cells specific for or delayed hypersensitivity to
M. tuberculosis antigens.
● This can be detected by either IFN-V release assays (IGRAs) or the tuberculin (purified
protein derivative [PPDI, or Mantoux) skin test.

S3
Question 75: Which of the following markers are typically expressed by neuroendocrine
tumours?
a. Synaptophysin
b. Chromogranin
c. CD56
d. CK7
R
Options:
A. Only c
EE
B. a,b, c
C. a, b, c, d
D. Only d
Correct option: B) a,b,c

Explanation:
● CK7 is an epithelial cell marker and is positive in carcinomas (especially the epithelial
R

tumours arising from the foregut). The following immunohistochemical markers are positive in
neuroendocrine neoplasms:
1. Synaptophysin
A

2. Chromogranin A
3. CD56
4. Neuron specific enolase (NSE).
C

Question 76: A 9-year-old boy with newly developed diabetes mellitus who died on the way to
hospital after a car accident will have what findings on autopsy suggest type 2 DM?

Options:
A. Amylin deposition in pancreatic islets
B. Armanni-Ebstein lesion
C. Insulitis
D. Kimmelstiel-Wilson nodules
Correct option: C) Insulitis.

Explanation:
● All the findings listed in the options are characteristic of both types of diabetes mellitus
except insulitis, which is specific to type 1 diabetes.
● Amylin deposition in pancreatic islets, derived from an insulin-associated polypeptide, is
found especially in type 2 diabetes mellitus.
● Armanni-Ebstein lesions (deposition of glycogen in renal tubules) are seen in

60
uncontrolled hyperglycemia, which can occur in either type.
● Kimmelstiel-Wilson nodules are seen in long-standing diabetes,
● Similarly, proliferative retinopathy is a complication of both forms of diabetes.

Question 77: A 41-year-old businessman presented to his primary care physician complaining

S3
of a purulent penile discharge. He has a history of unprotected sex with a woman he met at a
conference 1 week ago. Gram stains of the discharge fail to find any organisms. The most likely
cause of the discharge is?

Options:
A. Bowen disease
R
B. Chlamydia trachomatis
C. Herpes simplex virus
D. Treponema pallidum
EE
Correct option: B) Chlamydia trachomatis .

Explanation:
● Chlamydia trachomatis is one of the most common causes of urethritis, and it should be
considered if a gonorrhoea-like discharge lacks gram-negative diplococci inside neutrophilic
phagocytes. It's a sexually transmitted infection that can lead to epididymitis.
● A single erythematous plaque characterises Bowen's disease on the penis or scrotum,
R

which can progress to aggressive cancer.


● The herpes simplex virus can cause a vesicular rash on the penis. Syphilis is caused by
T. pallidum, which can manifest as a painless chancre on the penis.
A

Question 78: Type I membranoproliferative glomerulonephritis is commonly associated with:


1. SLE
2. Persistent hepatitis C infections
C

3. Partial lipodystrophy
4. Neoplastic diseases

Options:
A. 1,2,3
B. 1,2,4
C. 1,3,4
D. 2,3,4
Correct option: B) 1,2,4.

Explanation:
● MPGN Type 1:
● Characterised by subendothelial deposits.
● The basement membrane covers subendothelial deposits on both sides, which is
responsible for - the Tram Track's appearance.
● Partial lipodystrophy is associated with C3 nephritic factor (C3NeF)Type II MPGN.

60
Question 79:

S3
For the treatment of glaucoma, a patient was started on a drug. He came to the
R
ophthalmological outpatient department with complaints of his eyes appearing as shown.
Identify the drug's mechanism of action causing this finding as a side effect.

Options:
EE
A. Decrease aqueous production
B. Hyperosmotic agent
C.Decreased trabecular outflow
D. Increased uveoscleral outflow
Correct option: D)Increased uveoscleral outflow.
R

Explanation :
● Prostaglandin analogue - PGF2a analogue Latanoprost is the DOC in POAG.
● Mechanism:
Increases uveoscleral outflow, thus reducing IOP
A

● Adverse effects:
Hypertrichosis of eyelashes
Iris pigmentation
C

Herpetic ulcer
Conjunctival hyperaemia
Macular oedema.

Question 80:
A 5-year-old child is brought to the ophthalmology department with the following complaint.
Which of the following forms the first line of management for the patient?

Options:
A. Echothiophate eye drops
B. Echothiophate + Phenylephrine eye drops

60
C. Refractive correction
D. Orthoptic training
Correct option: C) Refractive correction.

Explanation:

S3
The image shows esotropia (convergent squint), commonly seen in children with hyperopia
(also known as hypermetropia). The first line of management in the treatment of esotropia
includes refractive error correction with glasses. Miotics are generally used for a child with
associated accommodation issues and non-compliance to glasses. Di-isopropyl
fluorophosphate (DFP) or echothiophate is commonly used for this condition. Long-term usage
causes secondary iris cysts. The concomitant use of phenylephrine can prevent these
R
secondary iris cysts. So, in such cases, both echothiophate + phenylephrine eye drops are
prescribed.
EE
Question 81: Drowsiness is caused by which of the following antiglaucoma medications?

Options:
A. Latanoprost
B. Timolol
C. Brimonidine
D. Dorzolamide
R

Correct option: C) Brimonidine.

Explanation:
A

● Brimonidine is selective a2 agonist agonist.


● Brimonidine causes drowsiness and is also a neuroprotective drug.
● Brimonidine has a dual mechanism of IOP lowering: it reduces aqueous humour
production and stimulates aqueous humour outflow through the uveoscleral pathway.
C

Question 82: Which of the following uveitic conditions is contraindicated to put an intraocular
lens after cataract extraction?

Options:
A. Fuch's heterochromic cyclitis
B. Juvenile rheumatoid arthritis
C. Psoriatic arthritis
D. Reiter's syndrome
Correct option: B) Juvenile rheumatoid arthritis.

Explanation:
Inflammatory reactions in children are always severe, and if the child is already suffering from
uveitis, IOL implantations can deteriorate the condition.

Question 83: A 55-year-old man complains that when he drives his car at night, he suffers from

60
glare in the eye, his best-corrected vision is 6112. What is the most probable diagnosis?

Options:
A. ARMD
B. Post capsular opacification lens

S3
C. Corneal degeneration
D. Diabetic retinopathy
Correct option: B) Post capsular opacification lens.

Explanation:
● Glare in the eye at night is due to posterior capsular opacification of the lens.

R At night - the pupil usually dilates →hence more light enters the eye → more light
creates more aberrations in the eye →more glare.
● Visual acuity in early posterior subcapsular cataracts can be 6/12.
EE
Question 84: Child with a mild squint. Intrauterine, birth history and developmental history to
date are all normal. Corneal reflex normal. Except for the exaggerated epicanthal fold, all other
eye parameters are normal. Diagnosis?

Options:
A. Accommodative squint
R

B. Pseudostrabismus
C. Esophoria
D. Exophoria
A

Correct option: B)Pseudostrabismus.

Explanation:
● Pseudoesotropia (apparent convergent squint )- Due to prominent epicanthal fold.
C

● Pseudoexotropia (apparent divergent squint) - Due to hypertelorism.

Question 85: Most common malignant tumour of the eyelid is?

Options:
A. Sebaceous gland carcinoma
B. Basal cell carcinoma
C. Squamous cell carcinoma
D. Malignant melanoma
Correct option: B) Basal cell carcinoma.

Explanation:
Basal cell carcinoma is the most common malignant tumour of the eyelids and constitutes
85-90% of all malignant epithelial eyelid tumours.

Question 86: How to differentiate between psychological and organic erectile dysfunction?

60
Options:
A. Nocturnal penile tumescence
B. PIPE therapy
C. Sildenafil-induced erection

S3
D. Squeeze technique
Correct option: A) Nocturnal penile tumescence.

Explanation:
One of the important methods to distinguish psychogenic impotence
from organic impotence is nocturnal penile tumescence & early morning erection, which are
R
preserved in psychogenic impotence but not in the organic cause of impotence.

Question 87: A Patient often falls with behavioural change and enuresis. What is the condition
EE
associated with him?

Options:
A. Frontotemporal dementia
B. Normal pressure hydrocephalus
C. Parkinson's disease
D. Alzheimer's disease
R

Correct option: B) Normal pressure hydrocephalus.

Explanation:
A

● Frontotemporal dementia - Associated with poor decision-making, language and


behavioural control.
● Normal-pressure hydrocephalus - Associated with gait disturbance, behavioural change,
enuresis, and dementia.
C

● Parkinson’s disease - Associated with tremors and many other muscle disorders like
dyskinesia.
● Alzheimer’s disease-Associated mainly with memory loss and confusion.

Question 88: A middle-aged man from West Bengal presents with paraesthesia of hands and
feet, hyperkeratosis of palms, lines on his nails and raindrop pigmentation. The most likely
causative toxin for the above-mentioned presentation is:
Options:
A. Lead
B. Arsenic
C. Thallium
D. Mercury
Correct Option: B) Arsenic.

Explanation:

60
Paraesthesia of hands and feet, hyperkeratosis of palms, lines on nails and raindrop
pigmentation are characteristic of arsenic poisoning.

Question 89: An alcoholic patient, the alleged assault by his neighbour, is brought for medical
examination. On examination - Irrelevant talk with abnormal behaviour is noted. He has a

S3
history of getting rid of alcohol for the past 3-4 days and has been diagnosed with delirium
tremens. According to the Indian Penal Code, which option is correct:

Options:
A. He is partially responsible for his act
B. He is criminally responsible for his act
R
C. Not responsible for his actions under section 84, IPC
D. Have diminished responsibility for his actions
Correct option: C) Not responsible for his actions under section 84, IPC.
EE
Explanation:
● Mc Naughten Rules are framed for the criminal responsibility of the insane. And it states
that an accused person is not legally responsible for crime if he/she is not of sound mind. It is
accepted in India as a law of criminal responsibility in section 84 IPC.
● In Mc Naughten's rule, insanity is a legal (not psychiatric) concept, and it judges delusion
as if real.
R

● Drunkenness (intoxication) is never an excuse against criminal responsibility unless it


becomes a disease (such as alcohol paranoia or delirium tremens). And then an act done by a
person of unsound mind, incapable of knowing the nature of the act due to intoxication, is not an
A

offence (section 84 IPC).

Question 90: A man fell from a height of 35 feet. Eyewitnesses say that he landed on his feet.
Which of the following injuries is possible?
C

Options:
A. Pond fractured skull with a cervical spine injury
B. Depressed fracture skull with a cervical spine injury
C. Gutter fractured skull with a cervical spine injury
D. Ring fracture of foramen magnum with a lumbar spine injury
Correct option: D) Ring fracture of foramen magnum with a lumbar spine injury.
Explanation:
RING FRACTURE:
● Commonly used to signify any fracture around the foramen magnum. Refers to fissured
fracture about 3.5 cm outside the foramen magnum at the back.
● It involves the middle ear laterally & roof of the nose anteriorly. Rare & requires
increased production of force. Causes:
● Fall from a height on feet or buttocks.
● Turn off the head-on spine suddenly.
● Severe blow on vertex driving skull downwards on the vertebral column.

60
● Heavy blow →underneath the occiput or chin.

Question 91: Ewing postulates concerns with which of the following:

Options:

S3
A. Growth at the site following trauma
B. Growth after a neurological injury
C. Age-related changes in the teeth
D. Old seminal stains
Correct option: A) Growth at the site following trauma.
R
Explanation:
Ewing's Postulates:
● Certain criteria, known as Ewing's postulates', must be satisfied before a relationship
EE
between trauma and new growth is accepted:
● The tumour must arise exactly at the site injured
● Definite and substantial trauma must be proved
● The tumour must be confirmed pathologically
● The tissue at the site must have been healthy before the trauma A reasonable
interval-neither too long or too short—must elapse between the time of the trauma and the
appearance of the tumour Though not one of Ewing's original postulates, there should be some
R

good scientific reason for ascribing the tumour formation to the injury, and this is rarely possible.
A

Question 92: An unconscious lady was brought from the village about 12 hours after ingesting
some kind of unknown poison. Her heart rate was 103/min, her blood pressure was 90/50 mm
Hg, and her respiratory rate was 19/min. Her breath smelled like kerosene. All of the following
should be done in her management, except:
C

Options:
A. Gastric lavage should be done
B. Atropine should be administered till signs of recovery
C. Vasopressors should be administered intravenously
D. Immediate airway management
Correct option: C) Vasopressors should be administered intravenously.
Explanation:
● Signs & symptoms suggestive of organophosphorus poisoning.
● The antidote of choice-Atropine.
● Reverts bradycardia & hypotension.
● Vasopressors contraindicated. Due to the cholinergic effect combated by atropine.

Question 93: All of these stains are used in tattooing except:

60
Options:
A. Osmium blue
B. Prussian blue
C. Vermillion
D. India ink

S3
Correct option: A) Osmium blue.

Explanation:
Osmium blue:
● Not used for tattooing.
● Tattoo marks:

R Design made in the skin by multiple puncture wounds with needles or an electric vibrator
dipped in colouring matter.
● Commonly used dyes:
EE
● Indian ink, carbon (black), cinnabar or vermilion (mercuric sulphide) red, chromic acid
(green), indigo, cobalt, Prussian blue (ferric ferrocyanide), ultramarine (blue).
Osmium tetroxide:
● Used in optical microscopy. Stains lipids.
● Dissolves in fats & reduced by organic materials to elemental osmium.
● Osmium- An easily visible black substance.
R

Question 94: Platauf's haemorrhages, incorrect is?

Options:
A

A. Sign of drowning
B.Subpleural haemorrhage
C. Mostly seen in the middle lobe
D. All are true
C

Correct option: C) Mostly seen in the middle lobe.

Explanation:
Paltauf's haemorrhages -
● They are subpleural haemorrhages in drowning cases when alveolar walls rupture due to
increased pressure during force in the expiration and produce haemorrhages.
● They are shining pale pink or bluish-red and maybe a minute or 35 cm in diameter seen
in the lungs.
● They are usually present in about 50 % of cases in the lower lobes of the lungs but may
be seen on the anterior surfaces of the lungs and the interlobar surfaces.

Question 95: Most common hymen rupture in a virgin

Options:
A. Anterior

60
B. Anterolateral
C. Posterolateral
D. Posterior
Correct option: C) Posterolateral.

S3
Explanation:
HYMEN RUPTURE
● Congenital : Anterior
● Due to intercourse or foreign body: posterolateral ( 4/8 or 5/7 0 clock) > posterior (6 0'
clock).

R In virgin, rupture (tears) of the hymen due to sudden stretching occurs in the posterior
half of the membrane, usually at the sides (i.e. posterolaterally) in the 4 or 8 O'clock or 5 or 7
O'clock position or in the midline of the hymen (60 clock position).
EE
● With first intercourse, tears usually occur in the posterior midline because the hymen lies
suspended across a potential space, whereas anteriorly periurethral tissues buttress the hymen.
More than 2 tears are unusual → semilunar hymen often rupture on both sides.
● Indicate of first sexual intercourse - Annular hymen, which nearly closes up the vaginal
orifice, may suffer several hymenal lacerations.
● One deep "V shaped cleft/tear at 6 O'clock or a number of clefts usually in posterior haft
hymen membrane indicate passage of any object through hyme orifice which is larger than its
R

original opening." The posterior tear may involve the fourchette producing a deep U-shaped
defect in prepubertal children. Fossa navicularis disappear, and the posterior commissure may
be ruptured due to torning of the fourchette. Until and unless there are many disproportions
A

between the male and female parts, the latter injury usually does not occur in consenting sexual
intercourse.

Question 96: Parents of the Child complain of assault by one of their relatives and anal pain in
C

a child, On investigation test, shows yellow iodine crystals with picric acid - what is the name of
the test?

Options:
A. Florence test
B. Barberio's test
C. Acid phosphatase test
D. Creatine Phosphokinase
Correct option: B) Barberio's test.

Explanation:
● It detects spermin (secreted by the prostate) and uses an aqueous or alcoholic solution
of picric acid.
● A saturated aqueous or alcoholic solution of picric acid, when added I to the spermatic
fluid, produces yellow needle-shaped rhombic crystals of spermine picrate. The reaction
depends on the presence of prostatic secretion.

60
● The Acid Phosphatase Test: The prostatic secretion element of seminal fluid contains
500 to 1000 times greater acid phosphatase than any other body fluid. Human red cells, semen
of higher apes, and juice of cauliflower have acid phosphatase levels similar to human semen.

S3
Question 97:

R
A 22-year-old male patient presents with features of bilateral conductive hearing loss not
associated with tinnitus. The given image shows the otoscopic findings of his tympanic
EE
membranes. What is this sign called?

Options:
A. Setting sun sign
B. Schwartze sign
C. Rising sun sign
R

D. Cart wheel appearance


Correct option: B) Schwartze sign.

Explanation:
A

● Schwartze sign -flamingo pink Tympanic Membrane.


● The otoscopic picture of both the tympanic membranes shows the Schwartze sign. This
finding, along with the history of bilateral conductive hearing loss, is seen in active otosclerosis.
C

● Schwartze sign -A reddish hue may be seen on the promontory through the tympanic
membrane →which indicates an active focus with increased vascularity.
● Tympanic membrane - It is quite normal and mobile.
● Rising sun sign - glomus tumour.
● Setting sun sign - Hydrocephalus.
● Cartwheel appearance - Acute suppurative otitis media.
Question 98: A 42-year-old male presents with left ear discharge and mild ear pain for the past
6 years. There is no history of hearing loss. On examination - Discharge is seen coming from
the posterior superior wall of the intact tympanic membrane. The tympanic membrane and the
left ear canal are normal. Most likely diagnosis?

Options:
A. Chronic suppurative otitis media
B. Chronic otitis externa
C. Keratosis obturans

60
D. Carcinoma of the external auditory canal
Correct Option: B) Chronic otitis externa .

Explanation:
Chronic Otitis Externa

S3
● Chronic otitis extrema is a chronic infection/inflammation involving the skin lining of the
external canal.
● Persistent low-grade infection/inflammation →leads to thickening of skin lining of the
external canal.
Aetiology:


R The primary cause - Repeated local irritation arising from persistent drainage from a
chronic middle ear infection.
EE
● Other causes - Repeated irritation, such as insertion of cotton swabs or other foreign
objects into the ear canal, can lead to this condition, as can rare chronic infections such as
syphilis and tuberculosis. and leprosy
Clinical Features:

● Unrelenting pruritus, mild pain


● The external canal shows the presence of dry skin.
R

● Asteatosis (lack of cerumen), hypertrophic external canal skin, presence of dry, flaky skin
in the external canal
● Mild tenderness on ear manipulation
A

● Rarely mucopurulent otorrhea Chronic Otitis Externa Management:


Treatment: Extensive use of acetic acid ear drops helps to reduce the pH of the skin lining the
external canal, making it more resistant to bacterial infections.
● Surgery is indicated in extreme cases. To widen the external canal, a canaloplasty is
C

performed. Replacement of involved skin by a split-thickness graft.

Carcinoma External Auditory Canal

● Most commonly, squamous cell carcinoma.


● Most commonly seen in cases of long-standing ear infections.
● It may arise primarily from the meatus or a secondary invasion from the middle ear
carcinoma.

Clinical Features

● Blood staining of hitherto mucopurulent or purulent discharge and severe headache

Examination:

60
● Ulcerated area in the meatus or a bleeding polypoidal mass or granulations
● Local extension of disease through the posterior meatal wall or its spread into the middle
ear may cause paralysis of the facial nerve .
● Regional lymph nodes (preauricular, postauricular, upper deep cervical and
infra-auricular )

S3
Treatment:
● En-bloc wide surgical excision with postoperative radiation.
● Keratosis Obturans: Collecting pearly white mass of desquamated epithelial cells in the
deep meatus.

Question 99: A 30-year female patient came with right-side hearing loss, better heard in a noisy
R
environment, Audiogram shown with about a 30-40 dB gap between AC-BC of the right & left
ear. Rinne's test was negative, and Weber's test was centralised. Which of the following
conditions is shown?
EE
Options:
A. Meniere's disease
B. stapedial otosclerosis
C. Presbycusis
D. Vestibular schwannoma
Correct option: B) Stapedial otosclerosis.
R

Explanation:
● In otosclerosis, the normally dense endochondral layer of the bony otic capsule gets
A

replaced by irregularly laid spongy bone. Most common site is fissula ante fenestram (anterior to
the oval window).
● The age group affected is 20-45 years (maximum between 20-30 years). Male: female
ratio is 1:2.
C

● Meniere's disease, i.e. Endolymphatic hydrops, leads to SNHL and not conductive
hearing loss.
● In otosclerosis, the tuning fork tests:
I)Rinnes test-negative
II)Weber's test lateralized to the ear with greater conductive loss.

Question 100: Which is the most common site for congenital cholesteatoma
Options:
A. Anterior superior quadrant of tympanic membrane
B. Posterior superior quadrant of tympanic membrane
C. Anterior inferior quadrant of tympanic membrane.
D. posterior inferior quadrant of tympanic membrane
Correct option: A) Anterior superior quadrant of the tympanic membrane.

Explanation:

60
● The posterior superior quadrant follows the anterior superior quadrant of the tympanic
membrane.
● Congenital cholesteatoma was defined as a white, pearly lesion behind an intact
tympanic membrane, which was identified on pathological examination as a cholesteatoma after
removal.

S3
● It is the more common cause for the onset of childhood conductive hearing loss
unrelated to middle ear effusion.

Question 101: 75-year-old diabetic patient presents with severe ear pain and granulation tissue
at the external auditory canal with facial nerve involvement. -The most likely diagnosis is:
R
Options:
A. Malignant otitis externa
B. Nasopharyngeal carcinoma
EE
C. Acute suppurative otitis media
D. Chronic suppurative otitis media
Correct option: A)Malignant otitis externa.

Explanation:
● The presence of a painful lesion in the external ear with the evidence of granulation
tissue and associated cranial nerve palsies (VIl nerve) in a diabetic (or immunocompromised)
R

patient suggest a diagnosis of malignant otitis externa.

Question 102: Systemic causes of epistaxis are all except:


A

Options:
A. Hypertension
B. Anticoagulant treatment
C

C. Hereditary telangiectasia
D. Hemophilia
Correct option: D)Hemophilia.

Explanation:

1. Epistaxis in Adult
● Primary
● Secondary

2. No cause is identified, but it may be due to,


● Use of NSAIDs
● Use of Alcohol
● Hypertension

3. The cause is identified, and it is due to:

60
● Coagulopathy secondary to liver disease/kidney disease/leukaemia or myelosuppression
● Trauma
● Post-surgery like inferior turbinectomy, Endoscopic sinus surgery
● Warfarin intake (anticoagulant treatment)
● Hereditary hemorrhagic telangiectasia

S3
● Haemophilia is a Secondary Cause of Epistaxis in Children.

Hence the answer is d.

4. Secondary epistaxis is not caused by haemophilia but by the aetiology of primary


epistaxis is implicated though its role is doubted there also.
R
Question 103: A 67-year-old person with hearing loss with normal speech discrimination is
suffering from?
EE
Options:
A. Noise-induced hearing loss
B. Presbycusis
C. Ototoxic drugs
D. NOHL
Correct option: B) Presbycusis .
R

Explanation:
● Information in this question is
A

(i) Old age (67 years),


(ii) Hearing loss, and
(iii) Preserved speech discrimination.
C

● The diagnosis is Presbycusis.


1. • Presbycusis refers to sensorineural hearing loss in elderly individuals
2. • Characteristically, presbycusis involves bilateral high-frequency deafness associated
with difficulty in central auditory processing information and speech discrimination.

● Four different pathological types of presbycusis have been identified those are :
1. Sensory presbycusis:-There is epithelial atrophy with loss of sensory hair cells and
supporting cells in the organ of Corti. This process starts in the basal turn of the cochlea and
slowly progresses towards the apex. Higher frequencies are affected, but speech discrimination
is preserved.
2. Neural presbycusis:- There is atrophy of nerve cells in the cochlea and central neural
pathways. Atrophy occurs throughout the cochlea (the basilar region is only slightly more
predisposed than the remaining part of the cochlea). Therefore, no precipitous drop in the
threshold of audiometry is observed. Speech discrimination is poor.
3. Metabolic (strial) presbycusis:- There is atrophy of stria vascularis. Atrophy results in

60
hearing loss represented by a flat audiogram, but speech discrimination is preserved.
4. Mechanical (cochlear conductive) presbycusis:- There is thickening and secondary
stiffening of the cochlea's basilar membrane. The thickening is more severe in the basal turn of
the cochlea, where the basilar membrane is narrow. This correlates with a gradually sloping
high-frequency sensorineural hearing loss. Speech discrimination is average.

S3
Question 104: A 10-year-old child presents with a non-foul purulent smelling discharge, which
is painless. The patient reports that he can hear better in the presence of discharge than when
the ear is dry. The most probable diagnosis is
Options:
A. CSOM
R
B. Serous otitis media
C. Cholesteatoma
D. Mastoiditis
EE
Correct option: A) CSOM

Explanation:
Clinical features of CSOM
● Profuse mucopurulent discharge which is not foul smelling, i.e... non-foul smelling
discharge.
● Hearing loss (conductive type). Its sensorineural component also occurs (i.e. mixed
R

type). It arouses the suspicion of toxic deafness. Sometimes, patients report a paradoxical
effect, i.e. hears better in the presence of discharge than when the ear is dry. This is due to the
round window shielding effect produced by a discharge which helps to maintain phase
A

differential.
● There is no pain; if it occurs, it is due to associated otitis externa, not due to otitis media.
● Since the infected area is open at both ends, the discharge does not accumulate in the
middle ear cavity.
C

● The ossicular chain is mostly uninvolved if involved, only a long process of the incus is
involved.

Question 105: Epistaxis after ligating the external carotid artery is due to which vessel?

Options:
A. Anterior ethmoidal artery
B. Superior labial artery
C.Sphenopalatine artery
D. Greater palatine artery
Correct Option: A) Anterior ethmoidal artery.

Explanation:
● Since the external carotid artery is ligated, the bleeding comes from branches of the
internal carotid artery.
● The anterior ethmoidal artery, a branch of the ophthalmic artery, a branch of the internal

60
carotid artery, is a constituent of the blood supply of the Little's area of the nasal septum.
● Blood supply of nasal septum
i) Internal carotid system-
● Anterior ethmoidal artery Branches of the ophthalmic artery
● Posterior ethmoidal artery

S3
ii) External Carotid System-
● The sphenopalatine artery (branch of the maxillary artery) gives nasopalatine and
posterior medial nasal branches.
● Septal branch of the greater palatine artery (Br. of the maxillary artery).
● R Septal branch of the superior labial artery (Br. of a facial artery).

Question 106: Popcorn calcification is seen in:

Options:
EE
A. Pulmonary hamartoma
B. Fungal infection
C. Metastasis
D. Tuberculosis
Correct option: A) Pulmonary hamartoma.

Explanation:
R

Popcorn calcification
● Popcorn calcification is a cluster of sharply defined, irregularly lobulated calcification,
usually in a pulmonary nodule.
A

● Hamartoma on chest X-ray


● The examination is the characteristic of popcorn calcification.
● Mediastinal lymph nodes of acute
● Histoplasmosis also shows this variant of X-Ray.
C

● Egg-shell calcification- Peripheral rim calcification of lymph nodes.


● It is seen in:

(i)N Silicosis (most common cause)


(ii) Histoplasmosis
(iii) Tuberculosis
(iv) Coal worker
(v)pneumoconiosis
(vi) Sarcoidosis
(vii)Coccidioidomycosis
(viii) Lymphoma following radiotherapy
(ix) Progressive massive fibrosis.

Question 107: Puff of smoke appearance on cerebral angiography is seen in:

Options:

60
A. ACA aneurysm
B. Cavernous sinus thrombosis
C. Moyamoya disease
D. Vein of Galen malformation
Correct option: C) Moyamoya disease.

S3
Explanation:
● Moyamoya disease is an idiopathic, non-inflammatory, non atherosclerotic progressive
vascular-occlusive disease involving the circle of Willis, most importantly the supraclinoid
internal carotid arteries.
● Small abnormal net-like vessels proliferate, giving the characteristic "puff of smoke"
R
appearance on direct angiography. Because of lower flow and spatial resolution, this
appearance is not always demonstrated by CTA and MRA.
EE
Question 108: A 52-year-old male presents with fever and malaise for 4 months and pain in the
knees and ankles. Blood tests are normal, apart from a raised ESR. Chest x-ray shows bilateral
hilar adenopathy, and pulmonary infiltrates, most severe in the upper and mid zones. The
Mantoux test is negative. What is the most likely diagnosis

Options:
A.Tuberculosis
R

B. Sarcoidosis
C. Asbestosis
D. Berylliosis
A

Correct option: B) Sarcoidosis.

Explanation:
● Sarcoidosis is the most likely diagnosis, given the presentation with malaise, arthralgia
C

and a chest x-ray showing bilateral hilar adenopathy.


● Radiological finding in patients with pulmonary sarcoidosis is characterised by bilateral
hilar lymphadenopathy.

Question 109: A 37-year-old factory worker developed itchy, annular scaly plaques in both
groins. The application of a corticosteroid ointment led to temporary relief but the plaques
continued to extend at the periphery. The most likely diagnosis is:
Options:
A. Erythema annulare centrifugum
B. Granuloma annulare
C. Annular lichen planus
D. Tinea cruris
Correct option: D) Tinea cruris .

Explanation:

60
● Tinea cruris is a dermatophyte infection of the groin.
● The presence of bilateral annular scaly plaques in the groin, itching and
unresponsiveness to steroids and peripheral extension leads to a diagnosis of Tinea cruris.
● The infection causes itching or a burning sensation in the affected area. Most likely
originating from the groin, thigh skin folds or anus.

S3
● Affected areas may appear reddish, tan, or brown, with flaking, rippling, peeling,
iridescence, or cracking skin.
● The acute infection begins with an area in the groin fold about a half-inch across, usually
on both sides.

Question 110: A 6-month-old child presented with diarrhoea for a few days. He had dermatitic
R
lesions on his hands and feet. Scalp hair was comparatively less. A deficiency of zinc was
suspected. TRUE about acrodermatitis enteropathica is:
EE
Options:
A. Autosomal recessive
B. Autosomal dominant
C. Zinc treatment partially cures
D. Treatment life long
Correct option: A) Autosomal recessive.
R

Explanation:
Acrodermatitis enteropathica (AE):
● It is an autosomal recessive disorder in which zinc absorption is defective.
A

● It presents in infancy, characterised by a triad of acral dermatitis (face, hands, feet,


anogenital area), alopecia, and diarrhoea.
● The human ZIP4 protein encoded on the SLC39A4 gene, an intestinal zinc transporter,
is defective in these infants.
C

● Mutations in this gene prevent appropriate enteral zinc absorption.


● Zinc bioavailability in AE, classically present during infancy on weaning from breast milk
to formula or cereal, is lower than in breast milk.

Question 111: Which Nerve biopsy is taken to diagnose neuritic leprosy?

Options:
A. Median Nerve
B. Ulnar Nerve
C. Radial Cutaneous Nerve
D. Sural nerve
Correct option: C)Radial Cutaneous Nerve

Explanation:
● Pure neuritic Hansen is characterised by sensory loss along the distribution of an
involved nerve trunk with or without motor deficit.

60
● In the absence of any Skin patch.
● The suitable nerves include the supra clavicular nerve, a supraorbital branch of the fifth
cranial nerve, the great auricular nerve in the neck, the cutaneous nerve of the forearm or thigh,
the radial cutaneous nerve at the wrist, the superficial peroneal nerve on the dorsum of the foot
or sural nerve at the back of the leg.

S3
● The nerves chosen for biopsy - are a branch of the radial cutaneous nerve at the wrist
region or a branch of the sural nerve at the level just above the ankle.

Question 112: A 37 years old male comes with complaints of baldness. well-defined bald
patches were seen with no scarring-on examination. Small broken hairs were seen in the
surrounding area. What is the likely diagnosis?
R
Options:
A. Androgenetic alopecia
EE
B. Alopecia areata
C. Anagen effluvium
D. Telogen Effluvium
Correct option: B) Alopecia areata

Explanation:
● Findings are highly suggestive of alopecia areata.
R

● The scalp appears normal in alopecia areata.


● Anagen is abruptly terminated in affected areas, and affected hairs move prematurely
into telogen, with resultant often precipitous hair shedding. The near pathognomonic
A

'exclamation point' hairs may be present, particularly at the periphery of areas of hair loss.
● Broader distal ends than the
● proximal ends of these short, broken hairs illustrate their inherent sequence of events:
follicular damage in anagen and a rapid transformation to telogen. White or greying hairs are
C

frequently spared and probably account, in fulminant alopecia areata, for the mysterious
phenomenon of 'going grey overnight'.

Question 113:
A 26-year-old man presented to the ER with a fever, splenomegaly, and a rash, as shown in the
image. After admission, he developed intestinal perforation, and an exploratory laparotomy

60
revealed several longitudinal ulcers in the small bowel. What is the probable diagnosis?

Options:
A. Enteric fever
B. Tuberculosis
C. Amoebiasis

S3
D. Hemorrhagic colitis
Correct option: A) Enteric fever

Explanation:
The given image shows Rose spots. This combination with fever, splenomegaly, longitudinal
R
ulcers in the intestine and intestinal perforation is suggestive of enteric fever. The term enteric
fever includes :

Typhoid fever-
EE
● Causative agent - Salmonella typhi and,
● Paratyphoid fever
● Causative agent - Salmonella paratyphi A, B and C.
● Enteric fever presents with step ladder pyrexia. The fever is accompanied by relative
bradycardia, the Faget sign. Hepatosplenomegaly is a common finding on examination. During
the second or third week, rose spots appear on the skin that fades with pressure. Since bile is a
R

good culture medium for the bacilli, they multiply abundantly in the gallbladder and are
discharged continuously into the intestine. In the intestine, it causes inflammation and necrosis
leaving behind the characteristic typhoid ulcers, which are oriented along the axis of the
intestine (longitudinal ulcers). Ulceration of the bowel leads to two major complications-intestinal
A

perforations and haemorrhage.

Question 114: A 46 years old male presents with itchy papules over his neck and V area of
C

chest and face for the last two and half years, which are exacerbated in summers and improved
in winters. What test will you do to confirm the diagnosis?

Options:
A. Patch test
B. Prick test
C. IgE levels
D. Skin biopsy
Correct option: A)Patch test

Explanation:
● The clinical picture mentioned above suggests a type of allergic air-borne contact
dermatitis.
● Most common source - Parthenium exposure
● Effect - It leads to dermatitis involving the exposed parts, i.e. Face, cubital fossa, upper
neck and popliteal fossa.
● The patch test is the only useful and reliable method for diagnosing allergic contact

60
dermatitis.

Question 115: Acanthosis nigricans is characterised by


all of the following except?

S3
Options:
A. Maybe an internal malignancy sign
B. Common in obese people
C. Associated with thick skin with hyperpigmentation
D. Histologically there is hypermelanosis
Correct option: C)Histologically, there is hyper melanosis.
R
Explanation:
● Acanthosis nigricans is brown to black discolouration, which usually
EE
affects body folds like the axilla, groin, umbilicus, and forehead.
● Acanthosis nigricans occur in individuals younger than 40 years of age. It is associated
with obesity (most common), endocrinopathies like insulin resistance DM, hypothyroidism,
Bloom syndrome, PCOD, and internal malignancy, e.g. gastric adenocarcinoma.
● Histopathologically, Papillomatosis is the characteristic feature, whereas no hyper
melanosis exists.
R

Question 116:
A
C

A cast was applied to a patient with a wrist injury, as shown in the image. What is this cast
known as?

Options:
A. Colles cast
B. Turn-buckle cast
C. Bachelor cast
D. Scaphoid cast
Correct option: D)Scaphoid cast.

Explanation:
● The cast applied to a patient in the glass-holding position is known as the scaphoid cast.
It is used to treat scaphoid fractures. To apply a scaphoid cast, the wrist is held in slight
dorsiflexion, and the thumb is in the abduction and slight flexion as if the glass is held between
the index finger and thumb. It extends from just below the elbow to proximal to the metacarpal
necks of the digits. On the thumb, the cast extends to just Proximal to the interphalangeal joint.

60
Scaphoid fracture presents a pain and swelling over the radial aspect of the wrist. A fall on an
outstretched hand could result in such a fracture.
● The fracture can be confirmed (when an X-ray doesn't show a fracture) if there is pain
elicited in the anatomical snuff box on the radial border of the wrist.
● An oblique x-ray of the wrist should be able to show a scaphoid fracture.

S3
Question 117: A 12-year-old boy presents with a symmetric, expansile cystic lesion in the
proximal humerus. All of the following can be done for his treatment except-

Options:
A. Curettage
R
B. Intralesional steroids
C. Intralesional sclerosing agents
D. Radiotherapy
EE
Correct option: B)Intralesional steroids.

Explanation:
● A solitary bone cyst is the diagnosis of a 12 years old boy who presents with a
symmetric, expansile cystic lesion in the proximal humerus.
Solitary Bone Cyst -
● Common in the first two decades, primarily between 5-15 years
R

● More common in males


● MC site: Proximal humerus (50-60%) >Femur (25-30%)
● The cyst appears as centrally radiolucent lesions on the metaphyseal side of the growth
A

plate of a long bone.


Treatment:
● Curettage
● Methylprednisolone acetate injection into the lesion
C

● Radiographic contrast injection into the lesion


● Intralesional injection of sclerosing agents
● Solitary Bone Cyst (Unicameral Bone Cyst) - Common in the first two decades, primarily
between 5-15 years.
● More common in males
● MC site: Proximal humerus(50-60%) >Femur (25-30%)
● It is not a true cyst because not lined by endothelial cells but lined by fibrous tissue and
blood vessels
Clinical Features:
● Asymptomatic unless presents as a fracture
● Radiological Investigations:
● The cyst appears as centrally radiolucent lesions on the metaphyseal side of the growth
plate of a long bone.
● The cortex is thinned but intact.
"Fallen fragment sign" may indicate that the lesion is fluid-filled rather than solid.

60
● Other tests are usually not required except in unusual locations such as the pelvis. In
such locations, MRI or CT is indicated.
● MRI can document the extent of the lesion and its cystic nature.
● MRI helps in distinguishing unicameral cyst (SBC) from aneurysmal bone cyst (ABC),
giant cell tumour (GCT) and fibrous dysplasia.

S3
Treatment:
● Curettage
● Methylprednisolone acetate injection into the lesion
● Radiographic contrast injection into the lesion
● R Intralesional injection of sclerosing agents.

Question 118:
EE
R

A 45-year-old lady suffering from hypothyroidism complains of pain and burning of the lateral
half of the palm. Her palms appear as shown on examination, and Phalen's test is positive.
What is the likely structure under which the affected nerve is compressed?
A

Options:
A. Ligament of Struthers
B. Bicipital aponeurosis
C

C. Guyon's canal
D. Flexor retinaculum
Correct option: D) Flexor retinaculum.

Explanation:
Like carpal tunnel syndrome, this image shows the atrophy of thenar eminence. It is due to the
median nerve, which passes under the flexor retinaculum in the wrist, is compressed. Patients
present with pain and paresthesia over the distribution of the median nerve. There may be a
weakness of thumb opposition. The thenar eminence is wasted in chronic cases. Tinel's sign will
be positive. This sign is elicited by percussing over the median nerve, reproducing symptoms. In
Phalen's test, the symptoms are reproduced by holding the wrists in full flexion for 60 seconds.
Carpal tunnel syndrome can be associated with rheumatoid arthritis, pregnancy, chronic renal
failure, acromegaly, hypothyroidism, and gout.

Question 119: An 8-year-old boy presents with swelling


in the mid-thigh region. On examination,
femoral thickening was found with

60
Codman's triangle. On aspiration, a greyish-white liquid showed atypical round cells with MIC-2
positivity. The most likely diagnosis is

Options:
A. Osteosarcoma

S3
B. Ewing's sarcoma
C. Tubercular osteomyelitis
D. Pyogenic osteomyelitis
Correct option: B)Ewing's sarcoma.

Explanation:


R
Ewing's sarcoma -
A type of primitive neuroectodermal tumour (PNET)
● Histopathological examination shows small round cells, which show positive
EE
immunohistochemical staining or CD-99 and MIC-2 genes.
● Although the classical periosteal reaction seen in Ewing's sarcoma is onion peel
appearance, sometimes it can present as Codman's triangle.
● Usually, Codman triangles and Sunray appearance are associated with osteosarcoma,
but they are also common in Ewing's
sarcoma. To diagnose osteosarcoma, histopathology must show the presence of immature new
bone cells, i.e. osteoid.
R

Question 120:
A
C

Find the correct statement regarding the x-ray shown below:

Options:
A. Low-velocity accidents/trauma are usually associated with this fracture
B. Odontoid fracture is the fracture of upper cervical vertebrae
C. Lower cervical spine fracture can be diagnosed with the x-ray view shown in the image
D. All of the above
Correct option: B) Odontoid fracture is the fracture of upper cervical vertebrae.

Explanation:
● The image shows an odontoid view or open-mouth view:
● Upper cervical spine fracture can be diagnosed with the X-ray view shown here.
● Usually, high-velocity accidents/ trauma are associated with this injury. Odontoid fracture
is the most common fracture of axis vertebrae (C2). Best investigation in NCCT. Classification:
Anderson D'Alonzo, with type 2 being the most dangerous.

60
Question 121: Marathon runners had pain in the anteromedial tibia on walking/jogging for long
hours. X-rays are normal. Doctor orders a bone scan. What may be the probable diagnosis

Options:

S3
A. Jones fracture
B. Shin splint
C. Lisfranc fracture
D. Nutcracker
Correct option: B) Shin splint.
R
Explanation:
● By definition, Shin splints (medial tibial stress syndrome) is an inflammation of the
tendons, muscles and bone tissue in and around the tibia.
EE
● Pain occurs along the inner border of the tibia, where muscles attach to the bone.
● The most common site of occurrence of a Shin splint is on the inside edge of your tibia.

Question 122: Von-Rosen's sign is positive in which of the following?

Options:
A. Perthes disease
R

B. SCFE
C. DDH
D. CTEV
A

Correct option: C) DDH .

Explanation:
Radiological features of DDH/CDH
C

In Von Rosen's view following parameters should be noted


1. Shenton's line: Smooth curve formed by the margin of the obturator foramen and inferior
border of the neck of the femur superior.

2. Hilgenreiner's line: Horizontal line drawn at the level of tri-radiate


cartilage
3. Acetabular index: Normally is 30°

4. CE angle of Wiberg: Normal value is 15-30°.


5. Perkin's line: Vertical line drawn at the outer border of the acetabulum

● The lower and inner quadrants are formed by two lines (Perkin's & Hilgenreiner's) where
the head lies In DDH, the head lies in the outer & upper quadrants.
● Shenton's line is broken.

60
● Retarded development and delayed appearance of ossification of the head of the femur.
● Sloping acetabulum.
● Superior & lateral displacement of the femoral head.
Von-Rosen's line:
● This is a line which helps in the diagnosis of DDH in infants less than 6 months.

S3
● For this, an AP view of the pelvis is taken with both lower limbs in 45°abduction and full
internal rotation.
● Upward prolongation of the long axis of the shaft of the femur points
● towards the lateral margin of the acetabulum and crosses the pelvis in the region of the
sacroiliac joint.
● In CDH, upward prolongation of this line points towards the anterior
R
superior iliac spine and crosses the midline in the lower lumbar region - Positive Von-Rosen's
sign.
EE
Question 123:
R
A
C

A patient was brought to the hospital with pain and swelling of the right hand and an inability to
move the fingers. The symptoms developed after he had punched a wall during a fight. X-ray
reveals a fracture, as shown by the arrow. The long bone fractured in the X-Ray is of which of
the following types?

Options:
A. Modified Long bone
B. Typical long bone
C. Miniature long bone
D. Irregular long bone
Correct option: C) Miniature long bone.

Explanation:
● 5th metacarpal is a miniature long bone
● Boxer's fracture
● Definition - Fracture of 5th metacarpal

60
● Aetiology- Punching against hard/ immovable object
● Clinical features -
1. Pain
2. Tenderness
3. Swelling

S3
4. Malalignment of finger
● Diagnosis -
X-ray AP and Lateral view of wrist-showing fracture of 5th metacarpal.
● Treatment -
1. Ice packs 2. Analgesics 3. Splinting.
R
Question 124: Fimbriectomy procedure is known as-

Options:
EE
A. Uchida method
B. Irving method
C. Madlener technique
D. Kroener method
Correct option : D) Kroener method.

Explanation:
R

● Uchida technique — In the mid portion of the tube, a saline solution is injected
subserosal to create a bleb.
● Irving method — The mid-portion of the tube (between the ties) is excised, and either
A

side is ligated.
● Madlener technique -It is the easiest method. An artery forceps are used for crushing the
loop of the tube.
● Uncommon procedures include -the Kroener method of fimbriectomy.
C

Question 125: A study was done to assess malnutrition among young children. 100 children
were selected from rural and urban areas. Out of these, 30 in rural and 20 in urban areas were
found to be malnourished. To compare the data sets, which statistical test is used?

Options:
A. Paired t-test
B. Chi-square
C. The standard error of the mean
D. ANOVA
Correct option: B) Chi-square test.

Explanation:
● Chi-square test :
It offers an alternative method for testing the significance of the difference between two
proportions.

60
● Advantage:
It can also be used when more than two groups are to be compared.

Question 126: If effective treatment for a disease is introduced in a community, what will affect
incidence [I] and prevalence [P]?

S3
Options:
A. No change in P & I
B. Both P & I will decrease
C. P will decrease & I will increase
D. P will decrease & I will remain the same
R
Correct option: D) P will decrease & I will remain the same.

Explanation:
EE
● New effective treatment will decrease the duration of the disease and thereby helps to
cure the patient.
● Incidence measures the rate at which new cases occur in a population; it is not
influenced by duration. So, new effective treatment will not affect incidence.
● On the other hand, prevalence will decrease due to a decrease in the duration of the
disease.
R

Question 127: According to female sterilisation 2014 guidelines, eligibility criteria for females
are sterilisation all except?
A

Options:
A. Age between 22-49 years
B. Should have at least 1 child
C. Unmarried woman
C

D. Partner is not sterilised


Correct option: C)Unmarried woman.

Explanation:
The eligibility criteria for female sterilisation (2014) are given as follows:
1. Clients should be ever-married.
2. Female clients should be above the age of 22 years and below the age of 49 years.
3. The couple should have at least one child whose age is above one
year unless the sterilisation is medically indicated.
4. To understand the full implications of sterilisation -Clients must be in a sound state of mind.
5. Clients or their spouses/partners must not have any history of sterilisation.
6. A relevant medical history, physical examination, and laboratory investigations must be
completed to ascertain eligibility for surgery.
7. Mentally ill clients must be certified by a psychiatrist, and a statement should be given by the
legal guardian/spouse regarding the soundness of the client's state of mind.

60
Question 128: All of the following are duties of an ASHA worker except:

Options:
A. Primary screening for the prevalence of non-communicable diseases
B. Assessing the success of national programs under ANM

S3
C. Administering zero dose of DPT and OPV
D. All
Correct option: B) Administering zero dose of DPT and OPV.

Explanation:
● ASHA doesn't receive financial remuneration for administering zero doses of DPT, and


R
OPV is not the function of ASHA.
ASHA Payments under Janani Suraksha Yojana (JSY): On the 45th Day:
I) In the case of institutional deliveries-6 visits (Day 3, 7, 14, 21, 28).
EE
II) In case of home deliveries -7 visits (Day 1, 3, 7, 14, 21, 28, 42).
● Birth weight record.
● Immunised with BCG, the first dose of OPV and DPT.
● Birth registration
● Mother & child are safe.
.
R

Question 129: Vertical transmission of HIV is highest with-


Options:
A. Elective cesarean section
A

B. High viral RNA load


C. Breastfeeding
D. Term delivery
Correct option: B) High viral RNA load.
C

Explanation:
● Vaginal and emergency cesarean section deliveries, prematurity, and low CD4 cell count
were most strongly associated with infant‚ infection status in univariate analyses.
● Children delivered vaginally or by emergency cesarean section were more likely to be
infected than those delivered by elective cesarean section, with a lower risk of 79% associated
with the latter (P<0.001).
● Similarly, infants delivered before 37 weeks were more than twice as likely to be infected
than infants who were not premature.
● The cesarean section before the onset of labour and rupture of membranes
approximately halves the risk of mother-to-child transmission.

Transmission rate:
● During pregnancy: 5–10%
● During labor and delivery:10–15%
● During breastfeeding:5–20%

60
● Overall without breastfeeding:15–25%
● Overall, with breastfeeding to six months:20–35%
● Overall, with breastfeeding to 18–24 months:30–45%

Question 130: People are separated into groups, from each group of people is selected

S3
randomly. What type of sampling is this -

Options:
A. Simple random
B. Stratified random
C. Systematic random
R
D. Cluster
Correct option: B) Stratified random.
EE
Explanation:
Separation of people into groups followed by random sampling from those groups are stratified
by random sampling.

Question 131: Secondary level of prevention is important in all of the following except?

Options:
R

A. Coronary heart disease


B. TB
C. Leprosy
A

D. None
Correct option: A)Coronary heart disease

Explanation:
C

For non-communicable diseases (e.g. CHD), primordial prevention is best intervention.

Question 132: Antiserum is available for passive immunisation against?

Options:
A. Typhoid
B. Rabies
C. Measles
D. Mumps
Correct option: B) Rabies

Explanation:
For passive immunisation
1. Antiserum
● Rabies
● Tetanus

60
● Diphtheria
2. Human normal immunoglobulins
● Rabies
● Tetanus
● Hepatitis A

S3
● Measles
● Mumps
3. Human specific immunoglobulins
● Diphtheria
● Hepatitis B
● Varicella.
R
Question 133: A 40-year-old female patient was brought to the ER with an altered sensorium.
On examination - BP: was 85/69mm with a pulse rate: of 65/min. The rectal temperature was 35
EE
Celsius. There was an associated history of dry skin, constipation and menorrhagia. What is the
diagnosis?

Options:
A. Septic Shock
B. Hypothermia
C. Myxedema Coma
R

D. Cardiogenic Shock
Correct option: C) Myxedema Coma.
A

Explanation:
Myxedema coma
● It is a state of decompensated hypothyroidism.
● Lab values of Myxedema coma are similar to a "normal" hypothyroid state.
C

● Still, a stressful event (such as an infection, myocardial infarction or stroke) precipitates


the myxedema coma, usually in the elderly.

The primary symptoms of myxedema coma are


1. Altered mental status
2. Low body temperature
3. Low blood sugar
4. Low blood pressure
5. Hyponatremia
6. Hypercapnia, hypoxia
7. Slowed heart rate and
8. Hypoventilation may also occur.

Question 134: A patient with Hb 6 was to be transfused with 2 packs of blood. The first pack
was transfused in 2 hours, after which vitals were stable. Transfusion of the next pack started
after that. Breathlessness and hypertension suddenly develop in the patient. This sudden

60
reaction is caused by which of the following?
Options:
A. transfusion-related circulatory overload (TACO)
B. transfusion-related acute lung injury (TRALI)
C. allergic reaction to transfused blood

S3
D. transfusion-related acute renal failure
Correct option: B) transfusion-related acute lung injury (TRALI).

Explanation:
Transfusion-related acute lung injury (TRALI)-
● It is a serious blood transfusion complication.

R It is characterised by the acute onset of non-cardiogenic pulmonary oedema following a
transfusion of blood products.
● Hypotension and fever develop within 6 hours after transfusion and usually resolve with
EE
supportive care within 48 to 96 hours. But hypotension is one of its most important diagnostic
features of it.

Question 135: A 15-year-old boy on a vegetarian diet presented with severe macrocytic
anaemia (haemoglobin- 5.1g/dL; mean corpuscular volume, 116fL) in addition to leukopenia and
thrombocytopenia (pancytopenia), icterus secondary to hemolysis and splenomegaly. Severe
vitamin B12 (cobalamin) deficiency. Following cobalamin replacement therapy, the patient
R

reported: Increased well-being, including weight gain, appetite and icterus resolved. Find the
false statement regarding the type of anaemia described here -
A

Options:
A. Reduced reticulocyte count
B. Macropolycytes
C. MCHC is increased
C

D. Hypercellular bone marrow


Correct option: C)MCHC is increased.

Explanation:
● Macropolycytes (large hypersegmented neutrophils) seen in megaloblastic anaemia
● MCHC is calculated by dividing MCH by MCV.
● Since both parameters [MCV, and MCH1 are increased proportionally in megaloblastic
anaemia, MCHC remains
● Thus, option 4 is wrong.
● The remaining features given in options can be seen in Megaloblastic anaemia.
● Note: An increase in MCHC occurs only in hereditary spherocytosis.

Question 136: Patient presented with PSVT after carotid management, BP is 70/30 mm/H &
vagal manoeuvre failed and refractory to adenosine given. what will be the next step-

60
Options:
A. Synchronised Cardioversion
B. Repeated adenosine
C. Inj. Amiodarone
D. DC shock

S3
Correct option: A) Synchronised Cardioversion.

Explanation:
● The next best step of management in this patient is synchronised cardioversion.
● The ECG suggests paroxysmal supraventricular tachycardia (PSVT) at a rate of
150/min. In this case, carotid massage and IV adenosine have failed to control PSVT, and the


R
patient continues to have hemodynamic instability (BP- 60/30 mmHg).
In this condition, synchronised direct current cardioversion (not just DC shock) should be
initiated with the starting dose of 50-100J.
EE
Question 137: A 59 years old male patient presented with complaints of right-sided facial
weakness and loss of sensation on the right side of the face with left-sided hemiparesis.
Examination reveals Horner's syndrome, nystagmus, vertigo, ataxia and dysphagia. It is also
observed that the patient loses his balance. What is the most common cause of this
presentation?
R

Options:
A. Due to occlusion of the right posterior inferior cerebellar artery
B. Due to occlusion of the basilar artery
A

C. Due to occlusion of the left posterior inferior cerebellar artery


D. Due to occlusion of the anterior inferior cerebellar artery
Correct option: A) Due to occlusion of the right posterior inferior cerebellar
artery.
C

Explanation:
● This case is highly suggestive of the lateral medullary syndrome.
Lateral medullary syndrome:
● Also referred to as “Wallenberg's syndrome”.
● Caused by occlusion of any of five vessels.
● Vertebral, posterior inferior cerebellar, superior, middle, or inferior lateral medullary
arteries.
● Embolic occlusion/thrombosis of ipsilateral V4 (4th segment of a vertebral artery)
segment → Lateral medullary ischemia.
● PICA occlusion also.
Features:
● The constellation of vertigo, numbness of the ipsilateral face & contralateral limbs,
diplopia, hoarseness, dysarthria, dysphagia & ipsilateral Homer's syndrome.
Partial syndromes:

60
● Due to occlusion of medullary penetrating branches of vertebral
● artery or PICA.
Hemiparesis:
● Not a feature of vertebral artery occlusion.
● Quadriparesis:

S3
● Due to anterior spinal artery occlusion.

Question 138: A patient with Hepatitis C exhibits hypo- complimentemia with 2 gm/day
proteinuria and hematuria. The most probable diagnosis is

Options:
R
A. MPGN
B. Cryoglobulinemia
C. Membranous glomerulopathy
EE
D. Post-infectious glomerulonephritis
Correct option: A) MPGN.

Explanation:
Type I MPGN shows:
● Persistent hepatitis C infections
● Autoimmune diseases like lupus or cryoglobulinemia, or neoplastic diseases. So, in the
R

above-mentioned patient with Hepatitis C, the most probable diagnosis is MPGN.


Membranoproliferative Glomerulonephritis(MPGN):
● MPGN is sometimes called mesangiocapillary glomerulonephritis or lobar
A

glomerulonephritis.
● It is an immune-mediated glomerulonephritis.
● The thickening of the GBM characterises it with mesangioproliferative changes; 70% of
patients have hypocomplementemia.
C

● MPGN is subdivided pathologically into 3 types - type I, type II, and type III
● disease.
● Type I MPGN is commonly associated with autoimmune diseases like cryoglobulinemia,
neoplastic diseases, lupus, and persistent hepatitis C infections.
● Except in patients with complement factor H deficiency, in the presence of C3 nephritic
factor and/or in partial lipodystrophy producing type II disease, or complement receptor
deficiency in type III disease, types II and III MPGN is usually idiopathic.
MPGN types :
Type I MPGN
● It is the most common MPGN.
● Idiopathic
● SABE
● SLE
● Hepatitis C ± cryoglobulinemia Mixed cryoglobulinemia
● Hepatitis b
● Cancer: Lung, breast and ovary (germinal)

60
Type lI MPGN(Dense Deposit Disease)
● Idiopathic
● C3, nephritic factor associated
Type Ill MPGN
● Idiopathic

S3
● Complement receptor deficiency.

Question 139: A patient on amphotericin B develops


hypokalemia of 2.3 meq/l. K+
supplementation required?
R
Options:
A. 60 mEq over 24 hours
B. 40 mEq over 24 hours
EE
C. 120-160 mEq over 24 hours
D. 80 mEq over 24 hours
Correct option: C) 120-160 mEq over 24 hours.

Explanation:
● Daily administration of potassium should not exceed 200 mEq.
● IV potassium is given to patients with severe hypokalemia who can not take oral
R

supplementation.
● Potassium may be given through a peripheral IV line in a concentration that should
exceed 40 mEq/L at rates up to 40 mEq/L/Hour in severe deficiency.
A

● The serum potassium levels should be checked every 3-6 hours, and Continuous ECG
monitoring should be done.
● For the initial administration, avoid glucose-containing fluid to
prevent the further shift of potassium into the cells.
C

● At the same time, Magnesium(Mn) deficiency must be corrected, particularly in refractory


hypokalemia.

Question 140: A patient with rheumatoid arthritis developed increased muscle tone of limbs
and sudden onset quadriparesis, Babinsky sign was positive with exaggerated tendon jerks and
worsening of gait with no sensation. sphincter involvement. The best initial investigation is
Options:
A. EMG and NCV within 48 hours
B. X-ray of the cervical area of the neck in flexion and extension
C. Carotid angiography
D. MRI brain
Correct option: B) X-ray of the cervical area of the neck in flexion and extension.

Explanation:
● Investigation to be done next is an X-ray of the cervical area of the neck in flexion and

60
extension in case of patients with rheumatoid arthritis with compressive Myelopathy or
neurological dysfunction.

Rheumatoid Arthritis:
● Neurological Involvement-Atlanto-axial involvement of the cervical spine is clinically

S3
noteworthy because of its potential to cause compressive Myelopathy and neurological
dysfunction.
● Neurological manifestations are rarely a presenting sign or symptom of atlanto-axial
disease, but they may evolve with
progressive instability of Cl or C2
● It does not affect the thoracic or lumbar spine.
R
Investigation:
● In patients with compressive Myelopathy or neurological dysfunction, the investigation to
EE
be done next is an X-ray of the cervical area of the neck in flexion and extension.

Question 141: A patient presents with ascending muscle weakness for 2 days. On examination,
the limb is flaccid. What investigation should be done first?

Options:
A. Serum potassium
R

B. Serum creatinine
C. Serum magnesium
D. Serum calcium
A

Correct option: A)Serum potassium.

Explanation:
● Hypokalemia can cause flaccid ascending paralysis, a differential diagnosis for GBS. It
C

can also cause periodic paralysis.


● Hypocalcemia and hypomagnesemia usually cause tetany, and hypercalcemia won't
cause ascending paralysis.
● A low serum potassium level during an attack, excluding secondary causes, establishes
the diagnosis of Hypokalemic Periodic Paralysis.

Question 142: For a 60-year-old patient who had a myocardial infarction 2 weeks back, the
lipid profile is done for a patient and reveals HDL 32 mg/dl, LDL 126 mg/dl, TG 276 mg/dl.
Which of the following is preferred for this patient?

Options:
A. Rosuvastatin + Fenofibrate
B. Fenofibrate alone
C. Rosuvastatin 10 mg
D. Atorvastatin 80 mg
Correct option: D)Atorvastatin 80 mg.

60
Explanation:
● When lipid-lowering agents are indicated, the treatment is started with IIMG Co-A
reductase inhibitors (statins). High-intensity statin therapy should be started in patients with
clinical atherosclerotic cardiovascular events (MI in this patient). In high-intensity statin therapy,

S3
either atorvastatin (40-80 mg/day) or rosuvastatin (20-40 mg/day) should be given.
● High-intensity Statin Therapy
● Lowers LDL cholesterol by approximately 50%.
Drugs used:
● Atorvastatin (40-80 mg)
● Rosuvastatin (20-40 mg).
R
Question 143: A 50-year-old male presented with complaints of ptosis, difficulty chewing and
occasional difficulty swallowing. There is no history of diplopia or visual
EE
loss. On examination, there is symmetric ptosis and mild restriction of extraocular
muscle movement with finger abduction test 60°. A nerve conduction study shows
the decremental response in orbicularis only. ERG revealed a myopathic pattern. Anti-AchR
radioimmunoassay was negative. The most probable diagnosis would be:

Options:
A. Ocular myasthenia gravis
R

B. Generalized myasthenia gravis


C. As anti-ACHR is negative, you will consider an alternative
diagnosis
A

D. Chronic progressive external ophthalmoplegia (CPEO)


Correct option: B) Generalized myasthenia gravis.

Explanation:
C

● Myasthenia gravis is a major cause of ptosis and diplopia.


● The distribution of muscle weakness has a characteristic pattern.
● The cranial muscles, particularly the lids and extraocular muscles, are often involved
early in the course, diplopia and ptosis are common
● initial complaints.

Question 144: For a 50-year-old patient who had a myocardial infarction 2 weeks back, the
lipid profile is done for a patient and reveals HDL 32 mg/dL, LDL 126 mg/dl, and TG 276 mg/dL.
Which of the following is preferred for this patient?

Options:
A. Rosuvastatin + Fenofibrate
B. Fenofibrate alone
C. Rosuvastatin 10 mg
D. Atorvastatin 80 mg
Correct option: D. Atorvastatin 80 mg

60
Explanation:
● Atorvastatin is an anti-dyslipidemic drug.
● Mechanism of action of Atorvastatin :
1. It inhibits HMG COA reductase enzyme- mevalonate is not formed from HMG COA

S3
cholesterol synthesis inhibited.
2. It helps to increase the number of LDL receptors in hepatocytes →more LDL is uptaken
by the liver.
3. Serum Cholesterol level decreased.
4. Statins have maximum LDL Cholesterol lowering potential.
5. Atorvastatin is a long-acting drug; that's why it can be administered at any time of the
day.
R
Question 145: A 20-year-old male presented with jaundice for the past 2 weeks. His liver
EE
function tests revealed serum bilirubin to
be 0.9 mg/dL and SGOT/SGPT to be 1240/1450 IU. He was positive for HBsAg and IgM
anti-HEV antibodies and negative for 1gM anti-HBc. The most likely diagnosis is

Options:
A. Acute hepatitis with mutant hepatitis B virus
B. Superinfection of hepatitis E virus with chronic hepatitis B
R

infection
C. Acute hepatitis with wild-type hepatitis B virus
D. Co-infection of hepatitis B and E viruses
A

Correct option: B)Superinfection of hepatitis E virus with chronic hepatitis B infection

Explanation:
● In the given question, the patient presents with jaundice, deranged LFT (raised
C

SGOT/SGPT), HBsAg positive, IgM anti-HEV antibodies positive and IgM antiHBc-negative. As
the patient is IgM anti-HBc-negative, acute hepatitis B is ruled out. As the patient is IgM
anti-HEV antibodies positive, the patient has acute Hepatitis E. In co-infection, both Hepatitis E
and Hepatitis B infections occur simultaneously.
● In the given question, hepatitis B is chronic, and hepatitis E is acute, so it favours the
superinfection of hepatitis E on hepatitis B.
Question 146: The tourniquet test is used in daily follow-up
of patients with:
Options:
a) Zika virus
b) Dengue virus
c) Chikungunya
d) Swine flu
Correct Answer: B)Dengue virus.

60
Explanation:
● A tourniquet test is used in the daily follow-ups of patients with the dengue virus.
● The tourniquet test (capillary-fragility test):
It is a clinical diagnostic method to determine a patient's hemorrhagic tendency, the fragility of
capillary walls and thrombocytopenia.

S3
Question 147: During methotrexate treatment, maintenance of high urinary pH is important
because:

Options:
A. Bladder irritation is reduced
R
B. Leucovorin toxicity ↑↑ in a dehydrated patient
C. Renal tubular secretion of methotrexate decreases
D. Methotrexate is a weak acid
EE
Correct option: D. Methotrexate is a weak acid

Explanation:
● Methotrexate is a weak acid
● It is reabsorbed in acidic urinary pH.
● Higher plasma concentration may result in toxicity.
● Therefore, high urinary pH must be maintained to decrease the reabsorption through
R

renal tubules and enhance its removal.

Question 148: Antipsychotic potency of the different compounds directly correlated positively
A

with an adverse effect of neuroleptic drugs. Which of the following is it?

Options:
A. Sedation
C

B. Postural hypotension
C. Extrapyramidal motor disturbances
D. Lowering of seizure threshold
Correct option: C) Extrapyramidal motor disturbances.

Explanation:
The antipsychotic potency of typical antipsychotic drugs is related to their D2 receptor-blocking
action. D2 blocking action is also responsible for extrapyramidal adverse effects like muscle
dystonia, parkinsonism hyperprolactinemia (dopamine acts like prolactin release inhibitory
hormone).

Question 149: A 13-year-old boy, on a vegetarian diet, complained of easy fatigability and
weakness. On further evaluation, he was found to be a case of Megaloblastic anaemia. Which
of the following is not a feature indicating megaloblastic anaemia?

60
Options:
A. Raised bilirubin
B. High reticulocyte count
C. Hypersegmented neutrophils
D. Pancytopenia

S3
Correct option: B) High reticulocyte count

Explanation:
● Reticulocyte count is low in megaloblastic anaemia d/t ineffective hematopoiesis.
● Please be clear that the reticulocyte count is increased in response to treating
megaloblastic anaemia with vitamin B12 and folate supplementation.


R The remaining features given in other options can be seen in megaloblastic anaemia.
Derangement in DNA synthesis causes most precursors to undergo apoptosis in the
marrow (an example of ineffective hematopoiesis) and leads to pancytopenia.
EE
● An accumulation of unconjugated bilirubin in the plasma due to the death of nucleated
red cells in the marrow (ineffective erythropoiesis).
● Neutrophils in peripheral blood are hypersegmented (more than five nuclear lobes).

Question 150: Inverted T waves are seen in

Options:
R

A. Hyperkalemia
B. Hyperthermia
C. Wellen syndrome
A

D. Coronary syndrome
Correct option: C) Wellen syndrome.

Explanation:
C

● Prominent T-wave inversions in the precordial leads occur in severe anterior wall
Ischemia (with or without infarction).
● High-grade stenosis of the left anterior descending coronary artery shows this pattern
(sometimes referred to as Wellens T-waves).
● Hyperkalemia has tall Tented T-waves.
● Coronary syndrome X is characterised by blockage of perforators, while the epicardial
coronary artery is normal. In these patients, stenting of coronaries is not useful. Nitrates are the
mainstay of therapy.

Question 151: A 23-year-old boy, a badminton player, sustained injury of the left ankle. He was
immobilised for 3 months, the cast was removed, and the patient could walk normally. Later, he
complained of pain and swelling in the left calf, ankle, and foot. His mother massaged him for 30
minutes. After a while, he developed an acute onset of breathlessness and was brought to the
emergency and died. The most likely cause of death is:

60
Options:
A. Pulmonary thromboembolism
B. Congestive cardiac failure
C. Massive stroke

S3
D. Hypovolemic shock
Correct option: A) Pulmonary thromboembolism.

Explanation:
A history of immobilisation for a long duration suggests a probably subclinical deep venous
thrombosis that had developed in the patient. On massaging, the thrombus of DVT got
R
dislodged in the pulmonary circulation, causing pulmonary thromboembolism and death.

Question 152: A 40-year-old patient complained of spikes of fever and difficulty breathing.
EE
Transesophageal ECHO found the vegetation in the heart. The culture was positive for
Burkholderia cepacia. The drug of choice for Burkholderia cepacia pneumonia is:

Options:
A. Aminoglycoside and colistin
B. Carbapenems with 3rd generation cephalosporins
R

C. Tigecycline and cefepime


D. Cotrimoxazole with 3rd generation cephalosporins
Correct option: D)Cotrimoxazole with 3rd generation cephalosporins.
A

Explanation:
● The drug of choice for B. cepacia is TMP-SMX, and alternative agents are ceftazidime,
chloramphenicol
C

● B. cepacia is the cause of a rapidly fatal syndrome of respiratory distress & septicemia
('cepacia syndrome')in cystic fibrosis patients.
● Predisposing factors: cystic fibrosis & chronic granulomatous disease
B. cepacia inhabits moist environments and is found in the rhizosphere.
possesses multiple virulence factors & colonising factors capable of binding to lung mucus
(predilection of B.cepacia for the lungs in cystic fibrosis).
● Treatment:
DOC for B. cepacia: TMP-SMX
● Alternative agents: Meropenem & doxycycline.

Question 153: Patient presenting with cutaneous vasculitis, glomerulonephritis, peripheral


neuropathy, Which investigation is to be performed next that will help you diagnose the
condition?

Options:
A. ANCA

60
B. RA factor
C. HBsAg
D. MIF
Correct option: A) ANCA.

S3
Explanation:
Anti-neutrophil cytoplasmic antibodies (ANCAs):
● Group of autoantibodies
IgG type is mainly produced against antigens in the cytoplasm of neutrophil granulocytes &
monocytes.
● It is called “ANCA- associated vasculitis” because it is particularly associated with
R
systemic vasculitis.

Question 154: A 50 years old male presented with frontal bossing, enlarged nasal bone, jaw,
EE
and spade-like fingers. Which of the following test will you do for diagnosis?

Options:
A. IGF1
B. ACTH
C. TSH
D. Serum cortisol
R

Correct option: A) IGF1.

Explanation:
A

● An elevated level of IGF-I provides a useful laboratory screening measure in case of


acromegaly
Diagnosis:
● Screening investigation of choice in acromegaly: IGF-l
C

● Confirmatory & Gold standard: Oral Glucose Tolerance tests.

Question 155: Double bubble sign is seen in :

Options:
A. Duodenal atresia
B.Duodenal stenosis
C. Volvulus
D. All of the above
Correct option: D) All of the above.

Explanation:
'a', 'b' & 'c' i.e. Duodenal atresia, Duodenal stenosis and Volvulus
Double bubble sign is seen in duodenal atresia, duodenal web, duodenal stenosis, Ladd's band,
Annular pancreas, Malrotation of gut, preduodenal vein.

60
Question 156: Abductor of Vocal cords is -

Options:
A. Lateral cricoarytenoid
B. Posterior cricoarytenoid

S3
C.Thyroarytenoid
D.Cricothyroid
Correct Option: B)Posterior cricoarytenoid

Explanation:
Vocal cords


R Abductors- Posterior cricoarytenoid
Adductors- Lateral cricoarytenoid
Thyroarytenoid
EE
Cricothyroid
Interarytenoid
● Tensors: Cricothyroid
Vocalis

Question 157: A 35-year-old man presented with on-and-off low back pain and early morning
stiffness for 2 years. X-ray spine showed squaring of the lumbar vertebra and erosion of the
R

sacroiliac joints. He is at an increased risk of developing all of the following:


Options :
A. Anterior uveitis
A

B. Inflammatory bowel disease


C. Hyperparathyroidism
D. Aortic regurgitation
Correct option: C)Hyperparathyroidism
C

Explanation:
● The above scenario is suggestive of ankylosing spondylitis.
● Hyperparathyroidism is not an extra-articular manifestation of ankylosing spondylitis.
● The extra-articular manifestations of ankylosing spondylitis are a consequence of
uncontrolled systemic inflammation.
They include:
● Eye- Anterior uveitis (commonest overall) , glaucoma.
● Renal – IgA nephropathy, secondary amyloidosis.
● Cardiac - Aortic regurgitation, aortitis, carditis, conduction defects.
● Respiratory- Upper lobe pulmonary fibrosis, pleural thickening Neurological - Cauda
equina syndrome.
● GIT- Inflammatory bowel disease.
● Diffuse osteoporosis.

Question 158: A 10 years old male presents a smooth swelling near the superficial inguinal

60
ring, which moves downwards when the testicle is pulled downwards. Diagnosis-

Options:
A. Inguinal hernia
B. Congenital hydrocele

S3
C. Encysted hydrocele of the cord
D. Varicocele
Correct option: C) Encysted hydrocele of the cord.

Explanation:
● A hydrocoele develops in a remnant of the processus vaginalis. somewhere along the
R
course of the spermatic cord. This hydrocoele also transilluminates and is known as an
encysted hydrocoele of the cord.
● The swelling moves downwards and becomes less mobile if the testis is pulled gently
EE
downwards.
● In females, a multicystic hydrocoele of the canal of Nuck sometimes presents as a
swelling in the groin.

Question 159: A middle-aged male complains of aches and numbness and sensation of fatigue
over his calf muscles that develop on exercise and are relieved on rest; this condition is not
associated with
R
A

Options:
A. Smoking
B. Hypocalcemia
C. Peripheral arterial disease
C

D. Hypertension and diabetes


Correct option: B) Hypocalcemia

Explanation:
The above condition may be associated with the following :
● Hyperlipidemia
● Diabetes
● Hypertension
● Cigarette smoking
● Alcohol, lipoprotein(a)
● Chlamydia pneumoniae
● Physical inactivity
● Herpes virus
● CMV infection.

Question 160: Gum tumour with 5 cm in dimension and contralateral lymph node enlargement

60
of 2 cm. There is no distant metastasis. The
stage of tumour:
Options:
A. T3N2M0
B. T2N2M0

S3
C.T1N2Mo
D. T3N3M0
Correct option: A) T3N2M0.

Explanation:
TNM STAGING OF ORAL CARCINOMA


R
Primary tumour, as follows:
T0 - No primary tumour
● Tis - Carcinoma in situ
EE
● T1 - Tumor 2 cm or smaller
● T2 - Tumor 4 cm or smaller
● T3 - Tumor larger than 4 cm
● T4 - Tumor larger than 4 cm and deep invasion to bone, muscle or
deep structures (e.g. antrum)

Lymphatic node involvement is as follows:


R

● N0: No regional lymph node metastasis


● N1: In a single ipsilateral lymph node, metastasis occurs.
● N2a: In a single ipsilateral lymph node >3 cm but not > 6 cm metastasis occur
A

● N2b: In multiple ipsilateral lymph nodes, none >6 cm in greatest dimension metastasis
occur
● N2c: In bilateral or contralateral lymph nodes, none 6 cm in greatest dimension
metastasis occur
C

● N3: In any lymph node >6 cm, metastasis occurs.


Tumor metastasis(M), as follows:
● M0 - No metastasis
● M1 - Metastasis noted
Staging
● Stage I : T1, N0, M0.
● Stage II : T2, N0, M0.
● Stage III : T3, N0, M0
● T1, T2, T3, N1, M0
● Stage IV : T4, N0, M0
● Any T, N2 or N3, M0
● Any T, any N, any M.

Question 161: The man presented with an acute abdomen; when the man was put in a
knee-chest position helped to relieve the pain, which might be the cause.

60
Options:
A. Acute pancreatitis
B. Cholecystitis
C. Superior mesenteric artery ischemia
D. Renal

S3
Correct option: A) Acute pancreatitis.

Explanation:
● Acute severe, refractory, upper abdominal pain radiating to back.
● By sitting or leaning forwards, some patients may gain relief.
● Biliary obstruction in gallstone pancreatitis can be Icterus.


R Grey turner's sign – bluish discolouration of the flanks
Cullen's sign – bluish discolouration around umbilicus
● Fox sign – discolouration below the inguinal ligament
EE
● Shock, acute renal failure, ARDS, MODS.
● Left-sided pleural effusion.

Question 162: Traumatic optic neuropathy due to close


head trauma commonly affects which part of the optic nerve -

Options:
R

A. Optic canal
B. Intra-ocular part
C. Intracranial part
A

D. Optic tract
Correct option: A) Optic canal.

Explanation:
C

Indirect optic nerve injury:


● If forces are applied to the malar eminences, and frontal bone →it gets transferred and
concentrated in the area near the optic canal. Within the optic canal, the tight adherence of the
optic nerve's dural sheath to the periosteum is also thought to contribute to this nerve segment
being extremely susceptible to the deformative stresses of the skull bones.
● Such injury leads to →retinal ganglion cells ischemic injury within the optic canal.
Question 163: Parathyroid glands are removed by surgery for recurrence investigation of
choice

Options:
A. Technetium scan
B. SPECT
C. MRI
D. Neck ultrasound
Correct option: A)Technetium scan.

60
Explanation:
Preoperative localisation tests for parathyroid glands are:
1. Tc 99m labelled sestamibi scan (>80% sensitive)
2. Single positron emission computed tomography (SPECT).

S3
3. CT and MRI scans.
4. Neck ultrasound.

Question 164: In Celiac artery compression syndrome, which structure is the main cause of
compression -
R
Options:
A. Median Arcuate Ligament
B. Rectus Sheath
EE
C. Deep Inferior Epigastric artery
D. Lacunar Ligament
Correct option: A)Median Arcuate Ligament.

Explanation:
● Median arcuate ligament syndrome (MALS, also known as celiac axis syndrome, celiac
trunk compression syndrome or Dunbar syndrome).
R

● Due to compression of the celiac artery and possibly the celiac ganglia by the median
arcuate ligament, abdominal pain occurs in this condition.
● It is also called celiac artery compression syndrome.
A

● The abdominal pain may be related to meals, accompanied by weight loss, and
associated with an abdominal bruit heard by a clinician.

Question 165: A 65-year-old male presenting with acute


C

pancreatitis now has refractory hypoxia. The X-RAY of the chest would show

Options:
A. Bilateral infiltrates
B. Pneumatoceles
C. Ground glass appearances
D. Hilar lymphadenopathy
Correct Option: A) Bilateral infiltrates.

Explanation:
Systemic complications of pancreatitis-
● Hypovolemic shock
● DIC
● ARDS
● Diabetes

60
Question 166: Preoperative prophylaxis for pheochromocytoma
Which of the following is required?

Options:
A. Alpha blockade after beta blocker

S3
B. Beta blockade after the alpha-blocker
C. Alpha blocker
D. Beta blocker
Correct option: B) Beta blockade after alpha-blocker.

Explanation:

R Beta-blockers such as propranolol at doses of 10 to 40 mg every 6 to 8 hours are often
needed in patients with persistent tachycardia and arrhythmias. n-blockers should only be
added after adequate a-blockade and rehydration.
EE
● In the immediate postoperative period, to prevent cardiovascular collapse and
postoperative hypotension, an adrenergic agonist is required.

Question 167: Perihepatic fibrosis occurring in Fitz Hugh Curtis Syndrome is due to -

Options:
A. Pelvic Inflammatory Disease
R

B. Bile Duct Injury


C. Chronic Alcoholism
D. Viral Hepatitis
A

Correct option: A)Pelvic Inflammatory Disease.

Explanation:
● Perihepatic fibrosis-
C

Direct intraperitoneal spread of infection towards the perihepatic region from initial pelvic
inflammation is thought to result in perihepatic fibrosis in Fitz Hugh Curtis Syndrome.
● Also, infection due to Trichomonas vaginalis, Ureaplasma urealyticum and Mycoplasma
hominis may also cause FHCS.
Question 168: A 45-year-old lawyer presents pain in the abdomen, more so in the epigastric
region, that worsens with eating spicy food and is relieved by bending forward. Complications of
the above-mentioned condition could be all except

Options:
A. Perforation
B. Bleeding
C. Gastric Outlet Obstruction
D. Splenic Vein Thrombosis

60
Correct option: D)Splenic Vein Thrombosis

Explanation:
Splenic Vein Thrombosis-
● Complications of peptic ulcer - bleeding, perforation and gastric outlet obstruction.

S3
● Perforation: This allows stomach contents to escape into the peritoneum, causing
peritonitis. It is more common in duodenal than in gastric ulcers.
● Gastric outlet obstruction: Ulcer near the pylorus is the most common cause of gastric
outlet obstruction, but the occasional cases are due to adult hypertrophic pyloric stenosis or
antral cancer.
● Bleeding.
R
Question 169: A 65-year-old male presents with CA prostate. The tumour is limited to the
capsule and palpable on PR examination. The patient is diagnosed as stage T1b. The best
EE
treatment would be -

Options:
A. Radical prostatectomy
B. Chemotherapy
C. Palliative radiotherapy
D. Orchidectomy
R

Correct option: A) Radical prostatectomy.

Explanation:
A

● T1b T1c, T2
● Radical prostatectomy or radiotherapy
● External beam radiotherapy is done for the T1 or low T2 disease and
● Brachytherapy is done for low T1 disease.
C

Question 170: A 22-year-old woman has had a non-progressive mass in the left breast for 6
months. There are no associated symptoms. Examination shows a mobile mass not attached to
the overlying skin or underlying tissue. The possible diagnosis is

Options:
A. Fibroadenoma
B. Cystosarcoma Phyllodes
C. Scirrhous Carcinoma
D. Fibroadenosis
Correct option: A) Fibroadenoma.

Explanation:
● Clinically, they present as firm, easily movable masses.
● MOUSE IN THE BREAST -They may be lobulated or smooth and slides easily under the
examining fingers.

60
● For discriminating between cysts and fibroadenomas, mammography is of little help;
however, ultrasonography can readily distinguish between them because each has specific
characteristics.
● For confirmation of the imaging findings: an FNA biopsy can also be used.

S3
Question 171:Which of the following is the best Stent for Femoropopliteal Bypass?
Options:
A. Dacron
B. Reversed saphenous
C. PTFE
D. None
R
Correct option: C)PTFE-covered self-expanding nitinol stents.

Explanation:
EE
● PTFE-covered stents are engineered with a 30–100-micron pore size.
Advantages of such pores:
● It allows healing for the endothelial lining of the stent-graft and vessel.
● For lower extremity bypasses, the two main grafts are the great saphenous veins and
polytetrafluoroethylene (PTFE) grafts.
● In treating chronic lower extremity ischemia, the most widely used stent-grafts is the
-Viabahn endoprosthesis.
R

Construction :
I) Expanded polytetrafluoroethylene (ePTFE) liner attached to an external nitinol stent.
II)The inner surface is bonded with heparin.
A

Question 172: A 25-year-old male has been receiving conservative management for an
appendicular mass for 3 days now and presents with a rising pulse rate, tachycardia and fever.
The mode of management must be -
C

Options:
A. Ochsner sherren regimen
B. Continue conservative management
C. Proceed to laparotomy and appendectomy
D. Intravenous antibiotics
Correct option: C) Proceed to laparotomy and appendectomy.
Explanation:
● Conservative Ochsner Sherren regimen is the standard treatment if an appendix mass is
present and the patient's condition is satisfactory.
● 10-20% need emergency operation due to spreading infection.

Question 173: A 40-year-old male with chest trauma presents with breathlessness, decreased
respiratory sounds on the right side, hyper resonance on percussion and distended neck veins.
The possible diagnosis is

60
Options:
A. Tension Pneumothorax
B. Cardiac Tamponade
C. Flail Chest
D. Myocardial Infarction

S3
Correct option: A) Tension Pneumothorax.

Explanation:
Cause of Tension pneumothorax :
● It occurs when the air becomes trapped in the pleural space under pressure.
● It develops as a 'one-way valve'.

R Leakage of air occurs either from the lung or through the chest wall.

Cause:
EE
● Blunt chest trauma, Penetrating chest trauma, Iatrogenic lung punctures (e.g. due to
subclavian central venepuncture) and Mechanical positive-pressure ventilation.

Clinical signs and symptoms -


● Dyspnea
● Tachypnea
● Hypotension
R

● Diaphoresis and
● Distended neck veins.
A

Question 174: Transplanted kidney is relocated to which region in the recipient's body?

Options:
A. Retroperitoneal region
C

B. Lumbar region
C. Epigastrium
D. Besides the dysfunctional Kidney
Correct option: A) Retroperitoneal region.

Explanation:
● Retroperitoneal iliac fossa - It is the most common place for placing a kidney transplant.
● In most cases, the Kidney is placed retroperitoneally, the iliac arteries and veins are
used for perfusion, and the ureter is transplanted directly into the bladder.

Question 175: In LAHSHAL terminology for cleft lip &


cleft palate, LAHSHAL denotes

Options:
A. Bilateral cleft palate only
B. Bilateral cleft lip only

60
C. Bilateral cleft lip & palate
D. No cleft
Correct option: C) Bilateral cleft lip & palate.

Explanation:

S3
● LAHSHAL classification of cleft lip and palate was proposed by
● Okriens .
● It is a diagrammatic classification of cleft lip & palate.

According to this classification, the mouth is divided into six parts:


● L-Right lip


R A-Right alveolus
H-Hard palate
● S-Soft palate
EE
● A-Left alveolus
● L-Left lip

Question 176: RTA with multiple fractures initial treatment would be-

Options:
A. Management of shock
R

B. Splinting of limbs
C. Airway management
D. Cervical spine protection
A

Correct option: C) Airway management.

Explanation:
● Management of trauma begins with a primary survey.
C

● ATLS (Advanced Trauma Life Support):


It defines a primary survey as an assessment of the `A, B, C', i.e.,
● A- Airway with cervical spine protection,
● B- Breathing and
● C- Circulation.
Question 177: 24-day neonate with projectile vomiting & failure to gain weight .what is the
diagnosis

Options:
A. CHPS
B. NEC
C. Duodenal atresia
D. Hirschsprung's disease
Correct option: A) CHPS.

60
Explanation:
● Projectile vomiting in 4th week indicates - CHPS.
● If vomiting (usually bilious)is right from the 1st day of life, then indicate duodenal atresia.
● On the other hand, NEC and Hirschsprung's disease have different clinical

S3
presentations.

Question 178: Injury to the penis which of the following prevents extravasation of blood –

Options:
A. Buck's fascia
R
B. Fascia of camper
C. Fascia transversalis
D. None
EE
Correct option: A) Buck's fascia.

Explanation:
Buck's fascia:
● Buck's fascia (deep layer of
● superficial fascia of the penis) superficial to tunica albuginea, a prolongation of colle's
fascia (membranous layer of the superficial fascia of the perineum).
R

● If Buck's fascia remains intact, the hematoma is restricted to the penile shaft, causing
eggplant deformity.
● Hematoma can extend to the scrotum,
A

● perineum and suprapubic regions if Buck's fascia is disrupted.

Question 179: RET proto-oncogene is associated with


C

the development of

Options:
A. Medullary carcinoma thyroid
B. Astrocytoma
C. Paraganglioma
D. Hurthle cell tumour thyroid
Correct option: A) Medullary carcinoma thyroid.

Explanation:
RET proto-oncogene:
● It is a tyrosine kinase-type growth factor receptor. The RET protein is a receptor for the
structurally related proteins, glial cell line-derived neurotrophic factor that promote cell survival
during neural development.

The following cells in which RET is normally expressed are:

60
● Parafollicular C cells of the thyroid and
● Adrenal medulla.
● Medullary carcinoma may occur in combination with adrenal pheochromocytoma and
hyperparathyroidism (HPT) (usually due to hyperplasia) in the syndrome known as multiple
endocrine neoplasia type 2A (MEN-2A).

S3
These are tumours of the parafollicular (C cells) derived from the neural crest rather than the
thyroid follicle cells, as are other primary thyroid carcinomas.
MEN type 2B syndrome:
● This syndrome occurs when the familial form is associated with prominent mucosal
neuromas involving the inner aspect of the eyelids, tongue and lips with a Marfanoid habitus.
RET associations:


R MEN 2a,
MEN 2b and
● Familial medullary carcinoma thyroid.
EE
Question 180: Misoprostol used in labour induction is an analogue of which of the following
type of prostaglandin?

Options:
A. PG E1
B. PG E2
R

C. PG I2
D. PGF2alpha
Correct option: A) PG E1.
A

Explanation:
Misoprostol is a methyl ester of PGE1.
Indications of Misoprostol:
C

● It is used for cervical ripening.


● Transvaginally it is used for induction of labour.
Uses of Prostaglandins in Obstetrics:
● Induction of abortion
● Termination of molar pregnancy
● Induction of labour
● Cervical ripening before induction of labour
● Acceleration of labour
● Management of atonic postpartum haemorrhage
● Medical management of tubal ectopic pregnancy.

Question 181: A mother brought her 16-year-old daughter to Gynaecology OPD with a
complaint of not attending menarche. She gives H/O cyclic abdominal pain. On further
examination, midline abdominal
swelling was seen. Per rectal examination reveals a bulging mass in the vagina. Which of the
following can be most commonly seen?

60
Options:
A. Imperforate hymen
B. Transvaginal septum
C. Vaginal agenesis

S3
D. MRKH
Correct option: A)Imperforate hymen.

Explanation:
● According to the given clinical scenario, patients suffer from primary amenorrhea and
cryptomenorrhea(cyclic abdominal pain).


R The girl is aged about 14–16 years.
Chief complaints -
i) periodic lower abdominal pain, which may be continuous,
EE
ii)primary amenorrhea and
iii)urinary symptoms, such as dysuria, frequency or urine retention.
● In significant cases, the presenting feature may be urine retention. The cause of
retention is due to the elongation of
the urethra.
● An abdominal examination reveals suprapubic swelling, which may be uterine or full
bladder. Prior catheterisation reveals the true state.
R

● Vulval inspection reveals a tense bulging membrane of bluish colouration.

Question 182: Emergency contraceptive should


A

be started within how much time after unprotected intercourse?

Options:
A. 24 hrs
C

B. 48 hrs
C. 72 hrs
D. 96 hrs
Correct option: C) 72 hrs.

Explanation:
● The morning-after pill : ethinyl-estradiol 2.5 mg, premarin 15 mg, the drug
is taken orally twice daily for 5 days.
● Beginning soon after exposure but not later than 72 hrs.

Question 183: Prolonged surgery time of vaginal hysterectomy would lead to damage to which
nerve ?

Options:
A. Obturator
B. Pudendal

60
C. Peroneal
D. Sural
Correct option: C) Peroneal.

Explanation:

S3
The most commonly injured lower extremity nerve in patients undergoing surgery in a lithotomy
position is the common peroneal nerve(LA-S2).

Question 184: A 21-year-old female presents to the emergency ward with 2 months of
amenorrhea with pain in the abdomen and shock. BP 90/60 mmHg and Hb 6 gm%.A urine
pregnancy test is found positive. The next immediate line of treatment is?
R
Options:
A. Laparotomy
B. IV fluids & cross match
EE
C. Medical management
D. Laparoscopy
Correct option: B) IV fluids & cross match.

Explanation:
● This is a case of ruptured ectopic pregnancy.
● A positive Urine Pregnancy Test suggests that amenorrhea is due to pregnancy.
R

● Due to ruptured ectopic →Pain and shock in early pregnancy occur.

Question 185: Patient with 3 months amenorrhoea, c/o


A

hyperemesis and vaginal bleeding for one month. On examination - uterus, 16.5 weeks with
absent fetal heart sound. The diagnosis is?

Options:
C

A. Vesicular mole
B. Ectopic pregnancy
C. IUFD
D. Abruptio placentae
Correct option: A) Vesicular mole.

Explanation:
Vesicular mole :
● It is an abnormal placenta with proliferative changes and hydropic degeneration in the
young chorionic villi.
● It is a benign condition with malignant potential.
● Uterine bleeding is almost universal.
● It may vary from spotting to profuse haemorrhage.
● It is the MC presenting feature:
● The discharge has a 'white currant in red currant juice' appearance.

60
Question 186: 27-year-old primigravida with 32 weeks of pregnancy suddenly complains of
oliguria, blurred vision, oliguria, and headache. Her BP is 180/110, and her urine albumin is +3.
The line of further management is?

Options:

S3
A. Wait And watch
B. LSCS
C. Induction of labour
D. Anticonvulsant + antihypertensive therapy
Correct option : D) Anticonvulsant + antihypertensive therapy.
R
Explanation:
● According to the clinical scenario, this is severe preeclampsia with impending eclampsia.
● Drug of choice( DOC) for eclampsia and impending eclampsia -Magnesium sulfate.
EE
Question 187: An infertile woman has a bilateral tubal block at cornua diagnosed with
hysterosalpingography. The next treatment of choice is?

Options:
A. IVF
B. Laparoscopy and hysteroscopy
R

C. Tuboplasty
D. Hydrotubation
Correct option: B)Laparoscopy and hysteroscopy.
A

Explanation:
Laparoscopy (with chromopertubation with methylene blue dye):
The best investigation for tubal patency, as tubal patency, can be confirmed under vision, and
C

besides, any pathology can be corrected with operative laparoscopy.

Question 188: Simultaneous administration of estrogen


and progesterone in hormone Does replacement therapy increase the risk of?
Options:
A. Ovarian cancer
B. Breast cancer
C. Cervical cancer
D. Both a and b
Correct option: D) Both a and b.

Explanation:
Risk due to both estrogen and progesterone in HRT:-
● Breast cancer
● Ovarian cancer
Risk due to only estrogen in HRT:-

60
● Endometrial carcinoma.

Question 189: Monozygotic twin with one healthy baby born at term and one dead mummified
fetus is suggestive of?
Options:

S3
A. Fetus acardiacus
B. Fetus papyraceous
C. Hydatidiform mole
D. Vanishing twin
Correct option: B) Fetus papyraceous.
R
Explanation:
Fetus Papyraceous or compresses:
● It is a state which occurs in the case of twins when one of the fetuses dies early.
EE
● The dead fetus is flattened, mummified and compressed between the membranes of the
living fetal and uterine walls.
● Both varieties of twins may show this condition, but it is more common in monozygotic
twins and is discovered at delivery or earlier by sonography.

Question 190: Following the criteria for conservative surgery in patients with ovarian carcinoma
except for?
R

Options:
A. FIGO stage II disease
A

B. Young patient with no or few children


C. Well-differentiated serous tumour
D. No infiltration of capsule, lymphatics or mesovarium
Correct option: A) FIGO stage II disease.
C

Explanation:
Requirements for conservative surgery in patients with
ovarian cancer is:
● FIGO stage IA disease
● Well-differentiated mucinous, serous, endometrioid or clear cell tumour.
● A young patient with no or few children.
● No other pelvic pathology precluding pregnancy.

Question 191: A 30-year-old is 14 weeks pregnant. She had two painless deliveries 16 weeks
earlier. The next line of management is?
Options:
A. Evaluation for thyroid disorders and diabetes mellitus (DM)
B. Cervical cerclage
C. Cervical length assessment
D. Tocolytics

60
Correct option: C) Cervical length assessment.

Explanation:
● In the past 16 weeks, the patient had two painless abortions →most probably, it is the
case of incompetent os.

S3
● The next line of management in these patients is frequent cervical length assessment:
clinically or by USG.
● The patient is evaluated more frequently, and if the cervix is short (less than 2.5cm), then
cervical cerclage has to be done.
● Cervical cerclage is the surgery of choice for incompetent os, but it can lead to
complications such as uterine contractions, abortions, and PROM. So if it is indicated, then only
R
surgery is to be done.

Question 192: If the fetus has hypoxia, which BPP parameter will be affected last?
EE
Options:
A. Fetal tone
B. Fetal breathing movement
C. Fetal movements
D. NST
Correct option: A) Fetal tone
R

Explanation:
The first activity to appear, Fetal Tone at about 7.5-8.5 weeks, is also
presumably, the last activity to disappear with progressively
A

worsening hypoxia.

Question 193: A 33 yr old female with heavy menstrual bleeding for 6 months comes to the
gynaecology OPD. On examination, no abnormality was seen. USG also appeared normal. The
C

patient tried to be managed with non-hormonal treatment, but it failed. What will be the next
management step?
Options:
A. Hormonal therapy
B. Endometrial sampling
C. Hysterectomy
D. Hysterectomy
Correct option: B)Endometrial sampling.

Explanation:
Endometrial sampling is done before starting the hormonal therapy as if hormonal therapy
precedes sampling, and the sampling results will be altered. As already mentioned in the
question, the USG and clinical examination show no abnormality hence the diagnosis can’t be
made on that so for diagnosis; we need to do endometrial sampling and see the hormonal
pattern. Then we can proceed with hormonal therapy.

60
Question 194: A 7-week pregnant lady has 1 accidental exposure to an x-ray. Which of the
following should be done?
Options:
A. Continue pregnancy
B. Terminate pregnancy

S3
C. Chromosome analysis
D. Pre-invasive diagnostic testing
Correct option: A) Continue pregnancy.

Explanation:
In the given case scenario, pregnancy will be continued as the exposure to X-ray radiation has a
R
very low range.
Exposure >15 rad during the second and third trimesters or >5 rad in the first trimester needs
patient counselling. Elective termination of pregnancy may be considered.
EE
Question 195: Which of these steps is followed first for
the management of shoulder dystocia after the McRoberts manoeuvre?
Options:
A. Sharp flexion of hip joints towards the abdomen
B. Supra pubic pressure
C. 90-degree rotation of the posterior shoulder
R

D. Emergency c-section
Correct option: B) Supra pubic pressure.
A

Explanation:
● Applying a gentle suprapubic pressure is the first step in managing shoulder dystocia
after the McRoberts manoeuvre.
Mnemonic “HELPERR” as a guide for treating shoulder dystocia:
C

● “H” stands for help


● “E” stands for evaluating for episiotomy.
● “L” stands for legs(pull your legs toward your stomach McRoberts
● manoeuvre.
● “P” stands for suprapubic pressure
● “E” stands for enter manoeuvres (internal rotation of baby’s shoulders)
● “R” stands for removal of the posterior arm from the birth canal.
● “R” stands for roll the patient.

Question 196: For uterine prolapse in pregnancy, can Ring Pessary be inserted up to?
Options:
A. 12 weeks
B. 14 weeks
C. 16 weeks
D. 18 weeks
Correct option: D) 18 weeks.

60
Explanation:
Pessary treatment for Prolapse:
● Stretching the hiatus genitalis it only gives symptomatic relief, it does not cure the
prolapse, thus preventing uterine & vaginal descent.

S3
● Indications of Pessary.
● Early Pregnancy - up to 18 weeks.
● Puerperium.
● Patients are unfit for surgery.
● Patients are unwilling to operate while waiting for surgery.
● Additional benefits like improvement of urinary symptoms.
R
Question 197: What are the signs of Hegar?
Options:
EE
A. Uterine contraction
B. Quickening
C. Bluish discolouration of the vagina
D. Softening of the isthmus
Correct option: D) Softening of the isthmus.

Explanation:
R

Hegar's sign
● Present in 2/3 of cases.
● Demonstrated in 6-10 week
A

Signs are based on the fad that:


1. The growing fetus enlarges the upper part of the body of the uterus.
2. the Lower part of the body is empty and extremely soft (just above the
C

cervix)
3. Cervixis comparatively firm.
Because of variation in consistent on bimanual examination (two
fingers in the anterior fornix and the abdominal fingers behind the
uterus) the abdominal and vaginal fingers seem to oppose below
the body of the uterus.
Question 198: A 27-year-old female with placenta previa
had severe bleeding. What is the most likely outcome post-delivery?
Options:
A. Galactorrhea
B. Diabetes
C. Absence of menstrual cycle
D. Cushing syndrome
Correct option: C) Absence of menstrual cycle.

60
Explanation:
Sheehan syndrome / postpartum hypopituitarism / postpartum pituitary necrosis :
● Due to blood loss and hypovolemic shock during and after childbirth →necrosis occurs,
which causes→hypopituitarism → Sheehan syndrome.
● The most common initial symptoms of Sheehan syndrome are agalactorrhea (absence

S3
of lactation) under difficulties with lactation.
● Many women also report amenorrhea or oligomenorrhea after delivery.

Question 199:
R
EE

This type of syringe is used for?


Options:
A. First trimester MTP
R

B. 2nd trimester MTP


C. Vacuum delivery
D. All of the above
A

Correct option: A) First trimester MTP.

Explanation:
● Manual vacuum aspiration (MVA) is the most effective and safe method of abortion in
C

which hand-held plastic aspirators are used to evacuate uterine contents.


● It is the treatment of choice for incomplete abortion for uterine sizes up to 12 weeks from
the last menstrual period (including
spontaneous abortion, miscarriage, and removal of retained products from an induced abortion),
endometrial biopsy and first-trimester MTP.

Question 200: If the anal sphincter is injured, which degree of Perineal Tear?
Options:
a) First
b) Second
c) Third
d) Fourth
Correct option: C) Third.

Explanation:
● Perineal Tears are classified into four categories, of which three of them are given below

60
:
● First-degree tear: laceration is limited to the superficial perineal skin, vaginal mucosa
and fourchette.
● Second-degree tear: laceration extends beyond the fourchette, vaginal mucosa and
perineal skin to perineal muscles and fascia, but not the anal sphincter.

S3
● Third-degree tear: fourchette, perineal skin, vaginal mucosa, muscles, and anal
sphincter are torn.

R END
EE
R
A
C

You might also like